Categories
Exam Questions Harvard

Harvard. Economics semester final examinations, 1900-01.

In the first full academic year of the twentieth century the Harvard economics department offered the following courses. The course links take you to the official course announcement, instructor names, enrollment figures, and the transcribed semester examinations.

Economics 1. Outlines of Economics
Economics 2. Economic Theory of the 19th Century
Economics 3. Principles of Sociology
Economics 5. Railways and Other Public Works
Economics 6. Economic History of the U.S.
Economics 8. Money
Economics 9. Labor Question in Europe and the U.S.
Economics 10. European Mediaeval Economic History
Economics 12. Banking and Leading Banking Systems
Economics 12a. International Payments and Gold/Silver Flows
Economics 13. Methods of Economic Investigation
Economics 17. Economic Organization and Resources in Europe
Economics 18. Principles of Accounting
Economics 19. General View of Insurance
Economics 20d. Adam Smith and Ricardo

__________________________________

Economics 1.
Outlines of Economics

Primarily for Undergraduates.

Course Announcement
  1. Outlines of Economics. , Wed., Fri., at 9. Professor Taussig, Dr. Sprague, Mr. Andrew, and Messrs. — and — .

Source: Harvard University. Announcement of the Courses of Instruction provided by the Faculty of Arts and Sciences for the Academic Year 1900-1901, p. 41.

Course Enrollment
  1. Professor [Frank W.] Taussig, Drs. [Oliver Mitchell Wentworth] Sprague and [Abram Piatt] Andrew, and Messrs. [Charles] Beardsley and [James Horace] Patten. — Outlines of Economics.

Total 442: 23 Seniors, 70 Juniors, 257 Sophomores, 29 Freshmen, 63 Others.

Source: Harvard University. Report of the President of Harvard College 1900-1901, p. 64.

1900-01
ECONOMICS 1
[Mid-year Examination]

Arrange your answers strictly in the order of the questions.

  1. In what manner do you think that (a) the individual efficiency of laborers, (b) their collective efficiency, would be affected by the general adoption of profit sharing? of socialism?
  2. It has been said that the original formation of capital is due to abstinence or saving, but its permanent maintenance is not. What do you say to either statement?
  3. Wherein is Walker’s presentation of the forces that make the general rate of wages better than Mill’s, wherein not so good?
  4. “The extra gains which any producer or dealer obtains through superior talents for business, or superior business arrangements, are very much of a similar kind. …All advantages, in fact, which one competitor has over another, whether natural or acquired, whether personal or the result of social arrangements, bring the commodity, so far, into the Third Class, and assimilate the advantage to a receiver of rent.”
    Explain (a) what is this Third Class, and what is the law of value applicable to it; (b) what Mill would say as to the proposition here stated; (c) what Walker would say?
  5. What qualifications of the general principle of rent can you state, in its application to (a) premises used for building purposes, (b) dwelling-houses, (c) mines?
  6. If all men had the same start in life, would there be differences of wages? If so, of what sort? If not, why not?
  7. “Since cost of production here fails us, we must revert to a law of value anterior to cost of production and more fundamental…” In what cases does cost of production fail us? Will “cost of reproduction” cover such cases? Is there another law more fundamental?
  8. Under what circumstances. if ever, will a general rise in wages affect the relative values of commodities? Would your answer be the same as to a general rise in profits?
  9. In what manner do you believe business profits, interest, and wages would be affected by the general adoption of the various forms of consumers’ coöperation? of producers’ cooperation?

Source: Harvard University Archives. Harvard University Mid-year Examinations, 1852-1943. Box 4, Bound volume: Examination Papers, Mid-Years, 1900-01.

1900-01
ECONOMICS 1
[Year-end Examination]

I.
Answer three.

  1. How will the value of land be affected by
    1. an increase in population,
    2. a reduction in the rate of interest,
    3. a protective tariff on agricultural produce.
  2. How will the price of grain be affected by
    1. a tax proportioned to the economic rent of the land,
    2. an equal tax upon all land.
  3. “Profits do not form a part of the price of the products of industry, and do not cause any diminution of the wages of labor.”
    Would Mill agree to this statement? Would you?
  4. Upon what does the general level of wages depend (a) according to Mill, (b) according to Walker? What would you expect these writers to say as to the effect of a protective tariff on the general level of wages?

II.
Answer two.

  1. If a country exports on a large scale a commodity not previously exported, will its other exports be affected? If so, how? If not, why not?
  2. Can a country have a permanently “unfavorable” balance of trade? If so, under what conditions? If not, why not?
    Can a country permanently export specie? If so, under what conditions? If not, why not?
    Can the rate of foreign exchange in a country be permanently at the specie-shipping point? If so, under what conditions? If not, why not?
  3. How would you expect the issue of a paper currency to effect foreign trade,—
    1. While the notes were still redeemable;
    2. After they had become irredeemable.

III.
Answer two.

  1. Define the following terms

Seignorage,
Clearing house loan certificates,
Silver Certificates,
United States notes,
Inconvertible paper.

  1. How would the adoption of bimetallism affect the stability of the value of money?
    1. according to Mill,
    2. according to Walker,
    3. in your own opinion.
  2. How is the value of money in a country likely to be affected by an increase in
    1. the quantity of commodities produced and sold,
    2. the quantity of bank notes,
    3. the volume of bank deposits.

Which of these changes would you expect to exercise most influence? Which least? Give your reasons.

IV.
Answer all.

  1. Compare and explain the operations of the Bank of England and those of the New York banks in a time of crisis,
  2. Arrange these items…

Government Securities 40.
Surplus 3.
Notes 38.
Specie 40.
Deposits 55.
Capital 14.
Loans 30.

    1. … in their proper order, as they would stand in an account of the Bank of France.
    2. … as they would stand in an account of a national bank of the United States; and state (1) whether this could be an account of a national bank, and (2) whether the proportions of the different items are such as you would be likely to find in an account of such a bank.
    3. … as they would stand in an account of the Bank of England, assuming the uncovered issue to be 17.

Source: Harvard University Archives. Harvard University, Examination Papers, 1873-1915. Box 5, Bound volume: Examination Papers, 1900-01. Papers Set for Final Examinations in History, Government, Economics, Philosophy, Education, Fine Arts, Architecture, Landscape Design, Music in Harvard College (June, 1901), pp. 21-23.

 

__________________________________

Economics 2.
Economic Theory
in the 19th Century

For Undergraduates and Graduates.

Course outline and readings.

Course Announcement
  1. Economic Theory in the Nineteenth Century. , Wed., Fri., at 2.30 Professor Taussig. [note: Professor Carver taught the course]

Source: Harvard University. Announcement of the Courses of Instruction provided by the Faculty of Arts and Sciences for the Academic Year 1900-1901, p. 41.

Enrollment
  1. Professor Carver. — Economic Theory in the Nineteenth Century.

Total 45: 6 Graduates, 15 Seniors, 16 Juniors, 5 Sophomores, 3 Other.

Source: Harvard University. Annual Reports of the President and Treasurer of Harvard College, 1900-01, p. 64.

1900-01
ECONOMICS 2
[Mid-year examination]
  1. Define value and explain why one commodity possesses more value in proportion to its bulk than another.
  2. Explain the various uses of the term diminishing returns, and define it as you think it ought to be defined.
  3. In what sense does a law of diminishing returns apply to all the factors of production.
  4. State briefly Böhm-Bawerk’s explanation of the source of interest.
  5. What, if any, is the relation of abstinence to interest.
  6. Would you make any distinction between the source of wages and the factors which determine rates of wages? If so, what? If not, why not?
  7. Discuss the question: Is a demand for commodities a demand for labor?
  8. What is the relation of the standard of living to wages.
  9. Discuss briefly the following questions relating to speculators’ profits. (a) Do speculators as a classmake any profits? (b) Are speculators’ profits in any sense earned?
  10. In what sense, if any, does the value of money come under the law of marginal utility?

Source: Harvard University Archives. Harvard University Mid-year Examinations, 1852-1943. Box 4, Bound volume: Examination Papers, Mid-Years, 1900-01.

1900-01
ECONOMICS 2
[Year-end Examination]

Discuss the following topics.

  1. The bearing of the marginal utility theory of value upon the questions of wages and interest.
  2. The definitions of capital as given by Taussig and Clark.
  3. Clark’s explanation of the place of distribution within the natural divisions of economics.
  4. Clark’s method of distinguishing between the product of labor and the product of capital.
  5. Clark’s distinction between rent and interest.
  6. Böhm-Bawerk’s theory of the nature of capital.
  7. The origin of capital, according to Böhm-Bawerk and Clark.
  8. The meaning of the word “productive” in the following proposition: “Protection is an attempt to attract labor and capital from the naturally more productive, to the naturally less productive industries.”
  9. The incidence of tariff duties.
  10. The theory of production and the theory of valuation as the two principal departments of economics.

Source: Harvard University Archives. Harvard University, Examination Papers, 1873-1915. Box 5, Bound volume: Examination Papers, 1900-01. Papers Set for Final Examinations in History, Government, Economics, Philosophy, Education, Fine Arts, Architecture, Landscape Design, Music in Harvard College (June, 1901), pp. 23-24.

 

__________________________________

Economics 3.
Principles of Sociology

For Undergraduates and Graduates.

Course Announcement
  1. The Principles of Sociology. — Development of the Modern State, and of its Social Functions. , Wed., and (at the pleasure of the instructor) Fri., at 1.30. Mr. —.

Source: Harvard University. Announcement of the Courses of Instruction provided by the Faculty of Arts and Sciences for the Academic Year 1900-1901, p. 41.

Enrollment
  1. Asst. Professor [Thomas Nixon] Carver. — The Principles of Sociology. Development of the Modern state, and of its Social Functions.

Total 57: 9 Graduates, 22 Seniors, 8 Juniors, 14 Sophomores, 4 Others.

Source: Harvard University. Annual Reports of the President and Treasurer of Harvard College, 1900-01, p. 64.

1900-01
ECONOMICS 3.
[Mid-year Examination]

Answer only ten questions.

  1. Upon what does Kidd base his argument that religion is necessary to keep men from taking such political action as would suspend economic competition, and what is the crucial point in his argument?
  2. In the light of Kidd’s theory of social evolution, discuss the question: Can there be a permanent civilization? Or, do the conditions which promote progress also ensure decay?
  3. Classify the sanctions for conduct which originate outside the individual and explain your classification.
  4. Explain and illustrate the meaning of the following: “Generalizing this struggle and extending it to every form existing in the social life — linguistic, religious, political, artistic, and moral, as well as industrial — we see that the really fundamental social opposition must be sought for in the bosom of the social individual himself.” (Tarde, Social Laws. Ch. II. p. 83.)
  5. What is meant by social stratification? How does it originate? What are some of its consequences?
  6. Compare Herbert Spencer’s theory of progress with Lester F. Ward’s, giving special attention to the argument which each offers in support of his theory.
  7. What, according to Patten, are the chief obstacles to a progressive evolution.
  8. Explain the following. “The difference between that society of conscious units which we call mind, and a society of human beings on our planet, is in the completeness of the mechanism.” (Patten, Theory of Social Forces. Ch. II. p. 21.)
  9. What, according to Patten, is the significance of the transition from a pain to a pleasure economy.
  10. How does Bagehot account for the origin of national traits?
  11. Discuss the question: Does charity retard the process of race improvement?

Source: Harvard University Archives. Harvard University Mid-year Examinations, 1852-1943. Box 4, Bound volume: Examination Papers, Mid-Years, 1900-01.

1900-01
ECONOMICS 3.
[Year-end Examination]

Discuss the following topics

  1. The definition of progress.
  2. Charity as a factor in human selection.
  3. The way in which, according to Spencer, the different classes of institutions are related to one another.
  4. The sanctions for conduct.
  5. A moral ideal as a factor in human selection.
  6. The natural antagonism of human interests and the problem of evil.
  7. The storing of the surplus energy of society.
  8. The influence of property on the relations of the sexes.
  9. Labor and service as bases of distributive justice
  10. The influence of militarism upon race development, or deterioration.

Source: Harvard University Archives. Harvard University, Examination Papers, 1873-1915. Box 5, Bound volume: Examination Papers, 1900-01. Papers Set for Final Examinations in History, Government, Economics, Philosophy, Education, Fine Arts, Architecture, Landscape Design, Music in Harvard College (June, 1901), p. 24.

 

__________________________________

Economics 5.
Railways and Other Public Works

For Undergraduates and Graduates.

Course Announcements

51 hf. Railways and Other Public Works, under Public and Corporate Management. Half-course (first half-year). Tu., Th. and (at the pleasure of the instructor) Sat., at 1.30. Mr. Meyer.

52 hf. Railways and Other Public Works (advanced course). Half-course (second half-year). Tu., Th. and (at the pleasure of the instructor) Sat., at 1.30. Mr. Meyer.

Source: Harvard University. Announcement of the Courses of Instruction provided by the Faculty of Arts and Sciences for the Academic Year 1900-1901, p. 42.

Enrollments

[Economics] 51 hf. Mr. [Hugo Richard] Meyer.— Railways and other Public Works, under Public and Corporate Management.

Total 86: 4 Graduates, 52 Seniors, 17 Juniors, 4 Sophomores, 9 Others.

[Economics] 52 hf. Mr. Meyer.— Railways and other Public Works (advanced course).

Total 9: 3 Graduates, 4 Seniors, 1 Junior, 1 Sophomore.

Source: Harvard University. Annual Report of the President of Harvard College, 1900-1901, p.64.

1900-01
ECONOMICS 51
[Mid-year Examination]

Omit the last question if the paper seems too long

  1. The construction put upon the long and short haul clause: by the Interstate Commerce Commission; by the Supreme Court.
  2. The decisions of the Interstate Commerce Commission on group rates.
  3. The railway rate situation in Germany [Prussia]; does it throw any light on the railway problem in the United States?
  4. “If pooling produces any beneficial result, it necessarily does so at the expense of competition. It is only by destroying competition that the inducement to deviate from the published rate is wholly removed….By the legalizing of pooling the public loses the only protection which it now has against the unreasonable exactions of transportation agencies.”—Give your reasons for accepting or rejecting this statement.
    Alternative:—
    The reasons for the instability of pools in the United States.
  5. The Iowa Railroad Commission.
    Alternative:—
    To what extent was the long and short haul clause of the Interstate Commerce Act enforced; what was the effect of that enforcement: on railway revenues; on intermediate shipping or distributing points?
  6. The body of administrative law to be found in the decisions of the Massachusetts Gas and Electric Light Commission’s decisions upon petitions for reductions in the price of gas.
  7. (a) Is it to the public interest to insert in street railway charters provisions seeking to secure to the municipality or the state a share in any excess of profit over the normal rate?
    Alternative: (b) and (c).
    (b) The evidence as to the return on capital obtainable in street railway ventures.
    (c) What questions of public policy were raised in the case of the Milwaukee Street Railway and Electric Light Co. vs. The City of Milwaukee?
  8. What statistics were used in illustrating in a general way the statement that railway charges are based upon what the traffic will bear; in discussing the bearing of stock-watering upon railway rates; in discussing the return obtained by capital invested in railway enterprises in the United States?

Source: Harvard University Archives. Examination Papers, 1873-1915. Box 5. Bound Volume: Examination Papers 1900-01. Papers Set for Final Examinations in History, Government, Economics, Philosophy, Education, Fine Arts, Architecture, Landscape Design, Music in Harvard CollegeJune, Pages 24-25.

1900-01
ECONOMICS 52
[Year-end Examination]
  1. The railways and the national finances in Prussia and Australia.
  2. Railway rates and the export trade of the United States since 1893, or, 1896.
  3. The economic situation in Australia since 1892, and the Australian railways.
  4. “A fatal objection to the income or preference bond is that it is an attempt to combine two contradictory commercial principles.”
    Discuss this statement fully. What does it mean? Is it true?
  5. If you had access to all the accounts of a railroad, how should you determine the value to it of one of its branch lines?
  6. To what accounts would you charge the following expenditures? (If you do not remember the exact Interstate Commerce Commission classification, use your best judgment.) State reasons in each case.
    Engineer’s wages on a special train conveying the general manager to an extensive flood covering the line.
    Fireman’s wages on an engine employed exclusively in switching to and from the repair shops.
    Conductor’s wages on a worktrain engaged in taking up rails on an abandoned branch.
    Brakeman’s wages on a train engaged solely in hauling company’s coal for company’s use.
    Cost of taking up comparatively new sound rails judged too light for heavy rolling stock.
    Cost at a competitive point of a new station to replace an old one which was large enough but old-fashioned.
  7. State the commonest problems facing a reorganization committee for an insolvent road, and then suggest and defend one course of procedure for each problem.
  8. Combine and arrange the following items so as to give the best information about the operation and condition of the road. (Do not rewrite the names but use the corresponding numbers where possible.)
1. Passenger train miles 2,000,000
2. Freight train miles 3,400,000
3. Passenger train earnings $2,400,000
4. Freight train earnings $5,500,000
5. Income from investments $100,000
6. Dividends $500,000
7. Operating expenses $4,700,000
8. Av. no. pass. cars per train 4
9. Av. no. passengers per car 11
10. Tons freight carried 2,800,000
11. Av. load per car (loaded and empty), tons 8.2
12. Av. no. loaded cars per train 12.3
13. Av. no. empty cars per train 6.7
14. Interest charge for year $2,200,000
15. Due other roads $100,000
16. Stocks and bonds owned $4,900,000
17. Supplies on hand $500,000
18. Taxes for the year $300,000
19. Accounts receivable $500,000
20. Cash $1,000,000
21. Surplus for the year $300,000
22. Profit and loss account $1,000,000
23. Taxes accrued but not due $100,000
24. Capital stock $50,000,000
25. Interest due $700,000
26. Funded debt $45,000,000
27. Due from other roads $100,000
28. Interest accrued not due $300,000
29. Franchises and property $90,400,000
30. Bonds of the company in its treasury $800,000
31. Accounts payable $1,000,000
32. No. of passengers carried 2,300,000

Source: Harvard University Archives. Examination Papers, 1873-1915. Box 5. Bound Volume: Examination Papers 1900-01. Papers Set for Final Examinations in History, Government, Economics, Philosophy, Education, Fine Arts, Architecture, Landscape Design, Music in Harvard College. June, 1901. Pages 25-27.

 

__________________________________

Economics 6.
Economic History of the U.S.

For Undergraduates and Graduates.

Course Announcement
  1. The Economic History of the United States. Tu., Th., at 2.30, and a third hour at the pleasure of the instructor. Mr. —.

Source: Harvard University. Announcement of the Courses of Instruction provided by the Faculty of Arts and Sciences for the Academic Year 1900-1901, p. 42.

Enrollment
  1. Professor Taussig. — The Economic History of the United States.

Total 164: 9 Graduates, 63 Seniors, 68 Juniors, 13 Sophomores, 11 Others.

Source: Harvard University. Annual Reports of the President and Treasurer of Harvard College, 1900-01, p. 64.

1900-01
ECONOMICS 6
[Mid-year Examination]

Arrange your answers strictly in the order of the questions. Answer all the questions,

  1. The nature and object of the scales of depreciation established by Congress and by the States at the close of the war of the Revolution; and how far these objects were accomplished.
  2. “The year 1789 marks no such epoch in economies as it does in political history.” — Taussig. How far is this true as to (1) financial legislation; (2) tariff legislation; (3) the course of foreign trade; (4) the growth of manufactures?
  3. Explain how you would distinguish Treasury Notes designed to circulate as currency from those designed simply to meet financial needs; and state when and under what circumstances, between 1789 and 1860, the United States resorted to issues of the first kind.
  4. Suppose the charter of the first Bank of the United States had been renewed: would the effect have been favorable or unfavorable for the finances of the government, for the bank, for the community, in 1812-1815?
  5. Suppose the charter of the second Bank of the United States had been renewed: would the effect have been favorable or unfavorable for the finances of the government, for the bank, for the community, in 1835-40?
  6. Describe the Independent Treasury system, as first established and as finally settled (give dates). Do you believe it better than the alternative system proposed by its opponents? Why?
  7. The causes of the crises of 1837 and 1857: wherein similar, wherein different.
  8. State what were the duties on cotton goods in 1809, 1819, 1839, 1859; and give your opinion whether the duties at these several dates were designed to give protection, and whether protection was then expedient.
  9. Why the early railway enterprises of the States were undertaken as public enterprises; and how far their history may be fairly cited for or against the policy of public management.

Source: Harvard University Archives. Harvard University Mid-year Examinations, 1852-1943. Box 4, Bound volume: Examination Papers, Mid-Years, 1900-01.

1900-01
ECONOMICS 6
[Year-end Examination]

Arrange your answers strictly in the order of the questions

  1. Explain summarily at what dates and to what extent land-grants and bond-subsidies were extended to railways by the United States; and state whether you believe these measures brought advantage to the country.
  2. Was the management of the finances during the Civil War fraught with more or less evil consequences than that during the War of 1812, as regards (1) the currency, (2) the banks?
  3. State what main sources of revenue were expected to be used, what were used in fact, by the United States in each of the years 1862, 1863, 1864; and explain how the resort to the sources actually used came about.
  4. For the decade 1870-80, explain the connection between the course of prices, foreign trade, railway operations, and currency legislation.
  5. For the decade 1880-90, connect the history of the public debt, the national revenues, the banking system, the silver currency.
  6. Does the argument for protection to young industries find support in the history of (1) the cotton manufacture before 1830, (2) the silk manufacture since 1870, (3) the tin plate industry since 1890.
  7. Explain how the theory of comparative costs may be applicable to the present situation as regards carpet wool, beet sugar, glassware, woollen cloths (take three).
  8. What changes were made in the duties on raw and refined sugar in 1890, 1894, 1897? Which mode of treatment do you regard the most advisable, and why?
  9. State what causes you believe to have chiefly promoted the growth and maintenance of the sugar and oil combinations; and consider which of these two you regard as typical, and as instructive for forecasting the future of combinations.

Source: Harvard University Archives. Examination Papers, 1873-1915. Box 5. Bound Volume: Examination Papers 1900-01. Papers Set for Final Examinations in History, Government, Economics, Philosophy, Education, Fine Arts, Architecture, Landscape Design, Music in Harvard College. June, 1901. Pages 27-28.

 

__________________________________

Economics 81
Money

For Undergraduates and Graduates.

Course Announcement

81 hf. Money. Half-course (first half-year). Tu., Th., Sat., at 11. Mr. Andrew.

Source: Harvard University. Announcement of the Courses of Instruction provided by the Faculty of Arts and Sciences for the Academic Year 1900-1901, p. 42.

Enrollment

81 hf. Dr. [Abram Piatt] Andrew. Money.

Total 122: 3 Graduates, 56 Seniors, 41 Juniors, 8 Sophomores, 1 Freshman, 13 Others.

Source: Harvard University. Annual Reports of the President and Treasurer of Harvard College, 1900-01, p. 64.

1900-01
ECONOMICS 8
[Mid-year Examination]

Answer only three questions from each group, but consider the questions strictly in the order of their arrangement 

I

  1. What is meant by

(1) a “double” standard;
(2) a “parallel” standard;
(3) a “limping” standard;
(4) a “single” standard;

Cite at least two historic examples of each, giving approximate dates.

  1. The following are estimates which have been made of the average production of silver, and its annual average export to the Orient in millions of ounces:

Production Export to East
1851-55 28 mill.

20 mill.

1855-60

29  ” 52  ”
1861-65 35  ”

53  ”

1865-70

43  ”

25  ”

State the causes of the singular situation revealed in these figures, and explain its actual effect upon the relative values of gold and silver in Europe.

  1. Suppose that the British government in 1870 had used the right conferred by the act of 1816, and had proclaimed the free coinage of silver at the ratio then current. What differences do you think would have occurred in the subsequent currency history of the world?
  2. Describe the effect of the suspension of the coinage of silver upon the value of the currency in each of the following cases:—
    (1) in Holland; (2) in Austria; (3) in Russia; (4) in India.

II

  1. “Before 1873 we had coined in the United States only about eight million silver dollars ($8,031,238) while since the date fixed as the beginning of demonetization we have coined nearly five hundred millions ($485,427,703).”
    How do you explain (1) the small amount of dollars coined before 1873? (2) the large amount coined since then?
  2. What in your opinion was the real significance of (1) the act of 1803? (2) the act of 1873?
  3. “With the exception of the brief period of fifteen years (1544-60) the English coins have never been debased.”
    In what sense and to what extent is this statement correct?
  4. In writing of the currency history of England during the years, immediately succeeding the great recoinage (1696) Mr. Dana Horton says:—
    “And so the full weight standard coin of the Realm, to create a stock of which the State had spent a sum greater than its regular annual revenue, and equal to perhaps a fourth of the country’s total stock of cash, — was allowed to find its way back to the melting-pot in exchange for cheaper gold.”
    Explain the situation to which he refers, and the reasons for this disappearance of the “standard coin.”

III

  1. (a) What were the main arguments which Lord Liverpool advanced in favor of a single gold standard?
    (b) What were the legislative acts in which his influence is to be traced?
  2. (a) Do falling prices necessarily mean an increase in the burden of debts?
    (b) Do they in the long run inevitably diminish the productiveness of industry?
    (c) To what extent are they prejudicial to the interests of the working classes?
  3. “It is generally agreed that every fall in the value of silver acted at the time as a stimulus to Indian exports and as a check on imports into India.”
    (1) Explain this statement, (2) state how far it is confirmed by commercial statistics, and (3) show whether such a condition is ever likely to be of prolonged duration.
  4. It is alleged that the Russian government, by stimulating exports, and hindering imports, has endeavored to secure a favorable balance of trade, with the idea of increasing the quantity of gold in the country? What do you think would be the ultimate effect of such a policy if continuously pursued?

Source: Harvard University Archives. Harvard University Mid-year Examinations, 1852-1943. Box 4, Bound volume: Examination Papers, Mid-Years, 1900-01.

Also: Harvard University Archives. Examination Papers, 1873-1915. Box 5. Bound Volume: Examination Papers 1900-01. Papers Set for Final Examinations in History, Government, Economics, Philosophy, Education, Fine Arts, Architecture, Landscape Design, Music in Harvard College (June, 1901), pp. 28-30.

 

__________________________________

Economics 9.
The Labor Question in Europe and the U.S.

For Undergraduates and Graduates.

Taught by W. F. Willoughby (Edward Cummings’ successor).

Course Announcement
  1. The Labor Question in Europe and the United States. — The Social and Economic Condition of Workingmen. Tu., Th., Sat., at 10. Mr. —.

Source: Harvard University. Announcement of the Courses of Instruction provided by the Faculty of Arts and Sciences for the Academic Year 1900-1901, p. 42.

Enrollment

92 hf. Mr. W. F. Willoughby. — The Labor Question in Europe and the United States. The Social and Economic Condition of Workingmen.

Total 146: 3 Graduates, 53 Seniors, 40 Juniors, 35 Sophomores, 3 Freshmen, 12 Others.

Source: Harvard University. Annual Reports of the President and Treasurer of Harvard College, 1900-01, p. 64.

1900-01
ECONOMICS 9
[Year-end examination]
  1. Show how the change in the organization of industry from the handicraft system and production on a small scale to the factory system and production on a large scale has led to; (a) efforts to supplant the wages system by socialism[,] coöperation, etc., (b) the trade union movement, and (c) compulsory compensation acts.
  2. Give the arguments for and against profit-sharing as regards (a) it being a more just system of enumeration than the wages system, and (b) its practical advantages.
  3. What are the two systems of coöperative production now practice in Great Britain, and why are they meeting with more success than earlier efforts?
  4. Describe the trade agreement between the National Metal Trades Association and the International Association of Machinists in such a way as to show its essential character and significance, and particularly its relation to the trade union movement and the question of the prevention and adjustment of industrial disputes.
  5. What was the nature of the “new unionism” movement in Great Britain, and its success?
  6. What is the general character of the Massachusetts State Board of Conciliation and Arbitration; what its duties and its powers?
  7. Describe the essential features of the French Workmen’s Compensation Act.
  8. Give a brief sketch of the Social Democratic Party in Germany, with the names of its early leaders and important events in its history.
  9. In what ways can the municipality take action for the improvement of the housing condition of the poorer classes without itself building tenements? What are some of the objections to the municipalities themselves undertaking building operations?
  10. Show why employment bureaus can do but little for the solution of the general problem of the unemployed.

Source: Harvard University Archives. Examination Papers, 1873-1915. Box 5. Bound Volume: Examination Papers 1900-01. Papers Set for Final Examinations in History, Government, Economics, Philosophy, Education, Fine Arts, Architecture, Landscape Design, Music in Harvard College (June, 1901), pp. 30-31.

Enrollment (Economics 9a)

9a2 hf. Mr. W. F. Willoughby. — Provident Institutions. Workingmen’s Insurance, Friendly Societies, Savings Banks.

Total 22: 1 Graduate, 13 Seniors, 5 Juniors, 2 Sophomores, 1 Other.

Source: Harvard University. Annual Reports of the President and Treasurer of Harvard College, 1900-01, p. 64.

1900-1901
ECONOMICS 9a
[Year-end Examination]
  1. What is the general situation in France at the present time in respect to insurance against old age and invalidity? Describe briefly the organization and workings of important institutions, and show particularly how the government is attempting to further this kind of insurance.
  2. What has been the general policy of the British government in respect to the regulation of Friendly Societies? Give the main features of law now regulating them.
  3. Describe the Fraternal Beneficial Orders of the United States as regards (a) their general scheme of organization, and (b) system of insurance.
  4. Show wherein this insurance system is defective by contrasting it with that of ordinary life insurance companies: indicate reforms that are necessary and how they can best be brought about.
  5. Contrast the systems of savings banks in Great Britain and the United States.
  6. In what respects are coöperative banks of the Schulze-Delitzsch and Raiffeisen type more valuable social institutions than the ordinary savings banks?
  7. Describe the principles upon which all coöperative building and loan associations in this country are organized, and indicate ways in which they might profitably be subjected to more rigid state control.
  8. Why is it impracticable to insure against unemployment?
  9. Outline briefly the system of sick insurance in Germany.

Source: Harvard University Archives. Examination Papers, 1873-1915. Box 5. Bound Volume: Examination Papers 1900-01. Papers Set for Final Examinations in History, Government, Economics, Philosophy, Education, Fine Arts, Architecture, Landscape Design, Music in Harvard College (June, 1901), p. 31.

 

__________________________________

Economics 10.
Mediaeval Economic History of Europe.

For Undergraduates and Graduates.

Course Announcement

For Undergraduates and Graduates.

  1. The Mediaeval Economic History of Europe. Tu., Th., and (at the pleasure of the instructor) Sat., at 12. Professor Ashley.

Source: Harvard University. Announcement of the Courses of Instruction provided by the Faculty of Arts and Sciences for the Academic Year 1900-1901, p. 41.

Enrollment
  1. Professor Ashley. The Mediaeval Economic History of Europe.

Total 11: 6 Graduates, 4 Seniors, 1 Junior.

Source: Harvard University. Annual Reports of the President and Treasurer of Harvard College, 1900-01, p. 64.

1900-01
ECONOMICS 10
[Mid-year Examination]

Not more than six questions should be attempted, of which the first should be one.

  1. Translate, and briefly comment upon
    1. Toto regis Willelmi primi tempore perseveravit haec institutio, usque tempora regis Henrici filii ejus; adeo ut viderim ego ipse quosdam qui victualia statutis temporibus de fundis regiis ad curiam deferri viderint.
    2. In Kateringes sunt X hidae ad geldum Regis. Et de istis X hidis tenent XL villani XL virgas terrae.
    3. Compotus Roberti Oldeman praepositi de Cuxham, ab in crastino Sancti Jacobi anno regni Regis Edwardi filii Regis Edwardi decimo usque ad in crastinum Sancti Jacobi proxime sequentis anno regni Regis Edwardi praedicti undecimo intrante.
    4. Rogamus . . . ademptum sit jus etiam procuratoribus nedum conductori adversus colons ampliandi partes agrarias aut operarum praebitionem jugorumve.
    5. Orgeterix ad judicium omnem suam familiam, ad hominum milia decem, undeque coëgit et omnes clientes obaeratoque suos quorum magnum numerum habebat eodem conduxit.
  2. What materials have we for forming a judgment as to the position of the rural population of England in the period from the eleventh to the fourteenth centuries? Classify them, and indicate the value of each class for the purposes of this enquiry.
  3. Wherein did the status of the coloni of the later Roman Empire resemble or differ from that of the medieval villein?
  4. Describe the constitution and working of manorial courts. What light does their history throw on the evolution of social classes?
  5. “Wie das Wort Dorf … dem Sinne nach einen Haufen bezeichnet, so ist auch haufenförmig oder Haufendorf der geeignetste Ausdruck für diese Art der Dorfenlage.” Explain and comment.
  6. “M. Fustel took for his point of departure the Provincial villa; Dr. Hildebrand takes the Kirghises of modern Asia.” Explain, and then show the peculiar dangers of each method.
  7. “We may safely follow Palgrave in taking the Anglo-Saxon townships as the integral molecules out of which the Anglo-Saxon State was formed.” Why? or why not?
  8. What was the gwely? What bearing has it on the general problem of “tribal” organization?
  9. What are the assumptions or postulates of modern Political Economy? To what extent were they true of the Middle Ages?
  10. Which book read in connection with this course has interested you most? Describe its method and estimate the value of its contribution to economic history.

Source: Harvard University Archives. Harvard University Mid-year Examinations, 1852-1943. Box 4, Bound volume: Examination Papers, Mid-Years, 1900-01.

1900-01
ECONOMICS 10
[Year-end Examination]

Not more than six questions should be attempted, of which the first should be one

  1. Briefly comment upon the following passages, and translate such of them as are not in English:—
    1. Colunt discreti ac diversi, ut fons, ut campus, ut nemus placuit. Vicos locant non in nostrum morem connexis et cohaerentibus aedificiis: suam quisque domum spatio circumdat.
    2. If a man agree for a yard of land or more, at a fixed rent, and plough it; if the lord desire to raise the land to him to service and to rent, he need not take it upon him, if the lord do not give him a dwelling.
    3. Ego S. … et ego P. … aliquantulam agri partem pro remedio animarum nostrorum W. episcopo in dominio donare decrevimus; id est xxx cassatorum in loco qui dicitur T.
    4. Si quis super alterum in villa migrare voluerit, et unus vel aliqui de ipsis qui in villa consistunt eum suscipere voluerit, si vel unus extiterit qui contradicat, migrandi ibidem licentiam non habebit.
    5. Qui habebant de tenentibus per diaetas totius anni, ut assolet de nativis, oportebat eos relaxare et remittere talia opera.
    6. If any one does an injury who is not of the gild and is of the franchise … he shall lose his franchise.
  2. Explain the position of Maitland’s Domesday Book and Beyond in the discussion concerning the origin of the manor.
  3. Distinguish between the several characteristics of mediaeval towns, and indicate the part played by each, in your opinion, in the formation of specifically urban conditions.
  4. Examine the relations between questions of personal status and questions of economic condition in relation to the ‘peasants’ of the Middle Ages.
  5. What is the nature of our evidence as to the Peasants’ Rising of 1381? Is there any reason for ascribing anything like an economic programme to the leaders of the movement?
  6. Indicate briefly (a), the several influences tending towards a corporate organization of industry in the later Middle Ages and (b) the advantages or disadvantages of such an organization.
  7. Distinguish between the several immigrations of foreign work people to England before the accession of James I, and explain the nature of their contributions to the development of English manufactures.
  8. The relation of John Major and Juan Vives to the development of the English ‘Poor Law.’
  9. What changes, if any, did the Reformation bring about in social life?

Source: Harvard University Archives. Examination Papers, 1873-1915. Box 5. Bound Volume: Examination Papers 1900-01. Papers Set for Final Examinations in History, Government, Economics, Philosophy, Education, Fine Arts, Architecture, Landscape Design, Music in Harvard College (June, 1901), pp. 32-33.

 

__________________________________

Economics 122.
Banking and the History of the Leading Banking Systems.

For Undergraduates and Graduates.

Course Announcement

122 hf. Banking and the history of the leading Banking Systems. Half-course (second half-year). Tu., Th., Sat., at 11. Dr. Sprague.

Source: Harvard University. Announcement of the Courses of Instruction provided by the Faculty of Arts and Sciences for the Academic Year 1900-1901, p. 43.

Enrollment

122 hf. Dr. [Oliver Mitchell Wentworth] Sprague. — Banking and the History of the Leading Banking Systems.

Total 128: 4 Graduates, 51 Seniors, 43 Juniors, 16 Sophomores, 14 Others.

Source: Harvard University. Report of the President of Harvard College, 1900-1901, p. 64.

1900-01
ECONOMICS 122
[Year-end Examination]

Arrange your answers strictly in the order of the questions. Answer all the questions under A and two of those under B

A

  1. Explain in detail and under different circumstances the effect of an advance of the rate of discount by the Bank of England upon the money market of London and upon the foreign exchanges.
  2. Taking the separate items of a bank account point out how those of the Bank of Amsterdam differed from those of a modern bank.
  3. Define and explain:—
    1. Bill broker.
    2. Banking Principle.
    3. The State Bank of Indiana.
    4. The banking law of Louisiana.
    5. Clearing House Certificates.
  4. The extent and banking consequences of government control of the Bank of France and the Reichsbank.
  5. How do government receipts and expenditures affect the money market (a) of London, (b) of New York?
  6. Explain with illustrations from the crises of 1857 and 1893 the nature of the demand for cash in time of crisis, and consider how far that demand may be met under a flexible system of note issue.

B

  1. (a) How far and with what qualifications may banking experience in the United States before 1860 be appealed to in the discussion of changes in our banking system? (b) How far, similarly, may Canadian experience be applied?
  2. “Why compel banks to send home for redemption a multitude of notes which can as well be used in payments and are sure to be reissued at once? Why impede the free use of its power of circulation by any enterprising bank by requiring the early redemption of notes which the holder does not in fact care or need to have redeemed?”
    Explain from past experience what regulations may be expected to bring about these results, and give the reasons for demanding them.
  3. Discuss the question of branch banking with reference to the United States, including in your discussion considerations of safety and economy. Would branch banking be more desirable than at present if notes were issued against general banking assets.

Source: Harvard University Archives. Examination Papers, 1873-1915. Box 5. Bound Volume: Examination Papers 1900-01. Papers Set for Final Examinations in History, Government, Economics, Philosophy, Education, Fine Arts, Architecture, Landscape Design, Music in Harvard College (June, 1901), pp. 34-35.

 

__________________________________

Economics 12a1.
International Payments and the Flow of Precious Metals.

For Undergraduates and Graduates.

Course Announcement

12a2 hf. International Payments and the Flow of the Precious Metals. Half-course (second half-year). Three times a week. Mr—.

Source: Harvard University. Announcement of the Courses of Instruction provided by the Faculty of Arts and Sciences for the Academic Year 1900-1901, p. 43.

Enrollment

[Economics] 12a1 . Mr. [Hugo Richard] Meyer.—International Payments and the Flow of the Precious Metals.

Total 16: 2 Graduates, 9 Seniors, 4 Juniors, 1 Other.

Source: Harvard University. Report of the President of Harvard College, 1900-1901, p. 64.

1900-01
ECONOMICS 12a1.
[Mid-Year Examination]

Observe strictly the order in which the questions are arranged.

  1. Sidgwick’s criticisms on Mill’s doctrine of international trade and their validity.
  2. What temporary changes in the general level of prices in this country should you expect to see, as the result of a large permanent withdrawal of foreign capital? What ultimate change of prices should you expect?
  3. Suppose the exportation of specie from the United States to be prohibited (or, as has sometimes been suggested, to be slightly hindered), what would be the effect on rates of exchange, and on prices of goods, either domestic or foreign? Would the country be a loser or not? [See Ricardo (McCulloch’s ed.), page 139.]
  4. The conditions which led to the flow of gold to the United States in the fiscal years 1880 and 1881?
  5. What economic conditions or events tended to make the year 1890 a turning point both in domestic and in international finance?

Alternative:

The reasons for the return flow from Europe of American securities in the years 1890-1900?

  1. What sort of wealth did France actually sacrifice in paying the indemnity? What was the process?
  2. Is Mr. Clare justified in making the general statement that “the gold-points mark the highest level to which an exchange may rise, and the lowest to which it may fall”?
  3. Why is it that certain trades bills are drawn chiefly, or even exclusively, in one direction, e.g. by New York on London and not vice versa; and how is this practice made to answer the purpose of settling payments which have to be made in one direction?

Alternative:

Why has England become the natural clearing-house for the world?

Source: Harvard University Archives. Harvard University Mid-year Examinations, 1852-1943. Box 4, Bound volume: Examination Papers, Mid-Years, 1900-01.

Also: Harvard University Archives. Examination Papers, 1873-1915. Box 5. Bound Volume: Examination Papers 1900-01. Papers Set for Final Examinations in History, Government, Economics, Philosophy, Education, Fine Arts, Architecture, Landscape Design, Music in Harvard College (June, 1901), pp. 33-34.

 

__________________________________

Economics 13.
Methods of Economic Investigation.

Primarily for Graduates.

Course Announcement
  1. Methods of Economic Investigation.—English Writers. German Writers. Tu., Th., at 1.30. Professor Taussig.
    Courses 15 and 13 are usually given in alternate years.

[15. The History and Literature of Economics to the close of the Eighteenth Century. Mon., Wed., and (at the pleasure of the instructor) Fri., at 12. Professor Ashley.
Omitted in 1900-01.]

Source: Harvard University. Announcement of the Courses of Instruction provided by the Faculty of Arts and Sciences for the Academic Year 1900-1901, p. 43.

Enrollment
1900-01

Economics 132 hf. Asst. Professor Carver. — Methods of Economic Investigation.

Total 10: 4 Graduates, 6 Seniors.

Source: Harvard University. Report of the President of Harvard College, 1900-1901, p. 64.

1900-01
ECONOMICS 13
[Year-end Examination]

Discuss ten of the following topics.

  1. The subdivision of economics into departments.
  2. The fields for the observation of economic phenomena.
  3. The place of historical and statistical research in economic investigation.

4, 5, 6. The methods of investigating:

    1. The causes of poverty.
    2. The effect of immigration on the total population of the United States.
    3. The effect of protection on the production of flax fibre, on the iron industry, or on any other industry which you may select.
  1. The nature of an economic law.
  2. The relation of the theory of probabilities to economic reasoning.
  3. The use of hypotheses in economic reasoning.
  4. The use of the terms “static” and dynamic in economic discussion.
  5. The use of diagrams and mathematical formulae in economic discussion.

Source: Harvard University Archives. Examination Papers, 1873-1915. Box 5. Bound Volume: Examination Papers 1900-01. Papers Set for Final Examinations in History, Government, Economics, Philosophy, Education, Fine Arts, Architecture, Landscape Design, Music in Harvard College (June, 1901), p. 35.

 

__________________________________

Economics 17.
Economic Organization and Resources of European Countries.

For Undergraduates and Graduates.

Course Announcement
  1. The Economic Organization and Resources of European countries. Mon., Wed., and (at the pleasure of the instructor) Fri., at 12. Professor Ashley.

Source: Harvard University. Announcement of the Courses of Instruction provided by the Faculty of Arts and Sciences for the Academic Year 1900-1901, p. 42.

Enrollment
  1. Professor Ashley. The Economic Organization and Resources of European countries.

Total 34: 5 Graduates, 14 Seniors, 9 Juniors, 3 Sophomores, 3 Others.

Source: Harvard University. Annual Reports of the President and Treasurer of Harvard College, 1900-01, p. 64.

1900-01
ECONOMICS 17
[Mid-year Examination]

Not more than eight questions should be attempted

  1. “It is less important for a particular community than ever it was to be in possession of cheap food and raw materials produced within its own domain.” Discuss this proposition.
  2. Describe very briefly the main features of the physical geography of England (illustrating your answer, if possible, with a map) and indicate in general terms their economic consequences.
  3. Set forth some of the general considerations which should be taken into account in answering the question whether the industrial development of Ireland has been injuriously affected by English legislation.
  4. Compare the number and character of the several classes maintained by agriculture in England, with those of the agricultural classes in the U.S. and on the continent of Europe.
  5. Explain the powers of dealing with his estate enjoyed at present by an English tenant for life under a settlement.
  6. What districts of England are now suffering most severely from agricultural depression, and why?
  7. Can any lessons be drawn for the U.S. from the recent history of productive coöperation in England? Give your reasons.
  8. Give a rapid survey of the apparent coal resources of the world.
  9. What points of especial interest are there to the economist in the history, situation, character, etc. of the South Wales Coal Field?
  10. What is meant by Collective Bargaining? What are its prerequisites? What examples of it are you acquainted with in America?

Source: Harvard University Archives. Harvard University Mid-year Examinations, 1852-1943. Box 4, Bound volume: Examination Papers, Mid-Years, 1900-01.

1900-01
ECONOMICS 17
[Year-end Examination]

Not more than eight questions should be attempted

  1. The British Chancellor of the Exchequer proposes to levy a duty of one shilling per ton upon the export of coal from the United Kingdom: He argues that the tax will not be borne by the producer, but mainly, if not wholly, by the foreign consumer. Consider (a) what are the conditions under which this is likely to be the case, (b) how far these conditions are at present realized in the case of England.
  2. Distinguish the successive stages in the technological history of iron and steel, and connect them with the industrial development of the several countries concerned.
  3. What were the questions at issue in England in the Engineering dispute of 1897? What, with your present knowledge, do you think ought to have been your attitude, had you then been (a) an English engineering employer, (b) a leading official of the employees’ union.
  4. Give a brief account of the organization of the English cotton manufacture (as distinguished from the securing either of the material or of a market for the product). Contrast it with American conditions; and consider how England and New England are likely to be affected by the growth of the manufacture in the Southern States.
  5. Distinguish between the several forms of capitalist combination at present to be observed in England. What general causes have led to the movement? What, if any, advantages does it promise, and what, if any, dangers does it threaten?
  6. Compare Bradford and Roubaix in any aspects which seem to you worthy of attention.
  7. “Lorsque il n’y a point d’hommes riches qui aient de gros capitaux à mettre dans les entreprises d’agriculture, lorsque les récoltes ne suffisent pas pour assurer aut entrepreneurs des profits égaux à ceux qu’ils tireraient de leur argent en l’employant de toute autre manière, on ne trouve point de fermiers qui veuilient louer les terres. Les propriétaires sont forcées de les faire cultiver par les métayers hors d’état de faire aucunes avances et de bien cultiver. Le propriétaire fait lui-même des avances médiocres qui lui produisent un très médiocre revenu.”
    Translate the passage from Turgot; and then consider how far his description applies to existing conditions in France and Italy.
  8. Show the relation of the great manufacturing industries of France to the distribution of coal in that country.
  9. Would the construction of the Rhine-Elbe canal be a benefit to Germany? Give your reasons.
  10. “Wir müssen uns Rechenschaft ablegen, ob ohne eine grössere Macht zur See, ohne eine solche die unsere Küsten vor Blockaden schützt, unseren Kolonialbesitz und unseren Welthandel absolut sicher stellt, unsere wirtschaftliche Zukunft gesichert sei.”
    Are there sufficient reasons in the contemporary situation of Germany for this anxiety on the part of Professor Schmoller?
  11. (a) Give a brief account of the contents, and then (b) compare the method and general attitude toward the subject, of von Schulze-Gaevernitz’s Social Peace and de Rousers’ Labour Question in Britain.
  12. What in the light of the experience in the English coal, iron, and cotton industries, would seem to you the most satisfactory form to be taken by joint wage agreements in the great industries of America?

Source: Harvard University Archives. Examination Papers, 1873-1915. Box 5. Bound Volume: Examination Papers 1900-01. Papers Set for Final Examinations in History, Government, Economics, Philosophy, Education, Fine Arts, Architecture, Landscape Design, Music in Harvard College (June, 1901), pp. 36-37.

 

__________________________________

Economics 18.
Principles of Accounting.

For Undergraduates and Graduates.

Course Announcement

181 hf. The Principles of Accounting. — Lectures, discussions, and reports. Half-course (first half-year). Mon., Wed., and (at the pleasure of the instructor) Fri., at 3.30. Mr. W. M. Cole.

Source: Harvard University. Announcement of the Courses of Instruction provided by the Faculty of Arts and Sciences for the Academic Year 1900-1901, p. 43.

Enrollment

181 hf. Mr. W. M. Cole. — The Principles of Accounting.

Total 56: 43 Seniors, 4 Juniors, 2 Sophomores, 7 Others.

Source: Harvard University. Annual Reports of the President and Treasurer of Harvard College, 1900-01, p. 64.

1900-01
ECONOMICS 18
[Mid-year Examination]

Problems 1 to 5 inclusive form a connected whole;
but 6
and 7 may be substituted for 4 and 5

I

  1. Construct a rough ledger (omitting rulings and index-memoranda) to correspond with the following trial-balance:
Real estate $150,000 Proprietor $244,275
Plant $60,000 Merchandise $401,000
Patents $40,000 Rent $6,000
Supplies $228,000 Bills payable $14,000
Wages $127,000 Accounts payable $43,000
Coal $9,000 Reserve fund $12,000
Insurance $4,500
Trade discounts $8,000
Interest $1,500
Bills receivable $10,000
Accounts receivable $68,000
Cash $14,275
$720,275 $720,275
  1. The above trial-balance is supposed to be taken from manufacturing books that are kept on the ordinary commercial plan, i.e., without distinctive accounts for stores, manufacturing, stock, or trading; and to construct such accounts now is supposed to be either impossible or undesirable.
    If you were required to determine profit and loss for the year which these figures cover, what questions about the business should you wish to ask before reaching your conclusions? [Give your answer in the form of questions consecutively numbered.]
  2. State what would be fairly reasonable answers to your own questions above numbering the answers to correspond with the questions; and then, assuming your answers to be the real answers show a complete statement of resources and liabilities and of profit and loss.
  3. Close for the year the ledger that you constructed indicating all balances that you have transferred to other accounts and all balances that you have carried down for the new year.
  4. From the ledger as it now stands draw off a balance sheet showing the condition of the business at the beginning of the new year, assuming that the loss or gain is carried directly to the proprietor’s account.
  5. Journalize the following:

A gives you his note for $100, bearing interest, dated a month ago.
You discount at a bank a note for $100 payable in a month
B gives you A’s note for $100 payable in one month, and buy goods for $100 on one month’s time.
Your book-keeper charged bills receivable and credited B when B paid his bill by your own note returned to you. A counter entry is to be made, so that the original wrong entry need not be erased

  1. What is the distinguishing feature of double entry? Are two postings made for every entry? If not, what devices are employed for reducing the number of postings?

II
Omit one

  1. The balance sheet of a corporation on January 1, 1899, stood as follows:
Real estate $50,000 Capital stock $200,000
Plant $95,000 Accounts payable $20,000
Horses, etc. $15,000 Bills payable $25,000
Patents $20,000 Profit and loss $15,000
Merchandise $30,000
Accounts receivable $30,000
Cash $20,000
$260,000 $260,000

On January 1, 1900, the books showed the following facts:

Real estate $55,000 Capital stock $200,000
Plant $88,000 Accounts payable $12,000
Horses, etc. $12,000 Bills payable $17,000
Patents $19,000 Profit and loss $33,000
Merchandise $42,000
Accounts receivable $28,000
Cash $18,000
$262,000 $262,000

What has become of the profits earned?

Should you recommend that a dividend be declared? State your reasons.

  1. How should you treat interest received on a bond bought above par?
  2. Describe the following, and state the distinguishing feature of each: a real account; a nominal account; a suspense account; reserve fund: a sinking fund
  3. If payments are received on account of goods in process of manufacture, should such payments appear on the balance sheet? If so, where?
  4. Describe three different methods of treating depreciation, and show how each would appear upon the books. To what circumstances on a railroad is each adapted?
  5. A corporation is formed to unite and continue the business of three concerns, A, B, and C, engaged in the same industry. The books of the concerns show the following:
A B C
Assets (valuation) $80,000 $160,000 $120,000
Liabilities (external) $20,000 $80,000 $90,000
Average profit, last three years 10% 14% 30%
Average profit, preceding three years 9 17 25
Average profit, prior three years 10 20 20

On what basis should you determine the total amount of capital stock to be issued by the new corporation, and on what basis should you apportion it to these three concerns?

Source: Harvard University Archives. Harvard University Mid-year Examinations, 1852-1943. Box 4, Bound volume: Examination Papers, Mid-Years, 1900-01.

 

__________________________________

Economics 19.
General View of Insurance.

Primarily for Graduates.

Course Announcement

192 hf. A General View of Insurance. — Lectures and reports. Half-course (second half-year). Mon., Wed., and (at the pleasure of the instructor) Fri., at 3.30. Professor Wambaugh.
Course 19 cannot be counted towards the degree of A.B.

Source: Harvard University. Announcement of the Courses of Instruction provided by the Faculty of Arts and Sciences for the Academic Year 1900-1901, p. 43.

Enrollment

192 hf. Professor Wambaugh. — A General View of Insurance.

Total 9: 6 Seniors, 3 Others.

Source: Harvard University. Annual Reports of the President and Treasurer of Harvard College, 1900-01, p. 64.

1900-01
ECONOMICS 192
[Year-end Examination]

One of the paragraphs may be omitted.

  1. From the point of view of the person procuring the policy, what is the purpose of insurance?
  2. From the point of view of the community, what are the advantages and the disadvantages of insurance?
  3. Give some account of three insurance books, pamphlets, or periodicals.
  4. Tell what you know of the history of insurance.
  5. Give a classification of the provision of the New York standard form of fire insurance policy,
  6. If either party to the fire insurance contract wishes to terminate the insurance, what are his rights?
  7. What are the benefits and the dangers of fire insurance by government?
  8. Describe ordinary life policies, single payment life policies, twenty payment life policies, endowment policies, tontine policies, assessment insurance.
  9. If a person thirty years of age wishes to obtain a life insurance policy for a single premium, how is the premium calculated?
  10. What are the chief differences between fire insurance and marine insurance?
  11. Discuss any insurance topic of which you have made a special study. 

Source: Harvard University Archives. Examination Papers, 1873-1915. Box 5. Bound Volume: Examination Papers 1900-01. Papers Set for Final Examinations in History, Government, Economics, Philosophy, Education, Fine Arts, Architecture, Landscape Design, Music in Harvard College (June, 1901), p. 40.

 

__________________________________

Economics 20d.
Adam Smith and Ricardo.

Primarily for Graduates.

Course Announcement

20d. Adam Smith and Ricardo. Half-course. Professor Taussig.

Source: Harvard University. Announcement of the Courses of Instruction provided by the Faculty of Arts and Sciences for the Academic Year 1900-1901, p. 43.

Enrollment

20d1 hf. Professor Taussig. — Adam Smith and Ricardo.

Total 12: 7 Graduates, 5 Seniors.

Source: Harvard University. Annual Reports of the President and Treasurer of Harvard College, 1900-01, p. 64.

1900-01
ECONOMICS 20d
[Final examination]
  1. Compare Ricardo’s conclusions with Adam Smith’s on the course of wages, profits, and rent, as society advances: discussing not only the conclusions themselves, but the reasoning by which the two writers arrive at them.
  2. Under what circumstances are real wages high, according to Adam Smith? according to Ricardo?
  3. Adam Smith’s doctrine on labor as a measure of value; Ricardo’s strictures thereon; and Ricardo’s own doctrine.
  4. S. Mill in his Autobiography says that “it was one of my father’s main objects to make me apply to Smith’s more superficial view of political economy the superior lights of Ricardo, and to detect what was fallacious in Smith’s arguments or erroneous in his conclusions.” Set forth how you believe the two Mills (father and son) set about this task as to Adam Smith’s reasoning on the following topics:—
    1. the mode in which the payment of heavy foreign obligations is brought about by the exportation of goods, not by the outflow of specie;
    2. the distinction between that land which always affords rent, and that which sometimes does and sometimes does not;
    3. the effect of foreign trade in raising the general rate of profits in a country.
  5. “That able but wrong-headed man, David Ricardo; shunted the car of Economic science on to a wrong line, a line, however, on which it was further urged by his equally able and wrong-headed admirer, John Stuart Mill.” — W. S. Jevons.
    What grounds are there for assenting to this judgment? What grounds for dissenting from it?

Source: Harvard University Archives. Examination Papers, 1873-1915. Box 5. Bound Volume: Examination Papers 1900-01. Papers Set for Final Examinations in History, Government, Economics, Philosophy, Education, Fine Arts, Architecture, Landscape Design, Music in Harvard College (June, 1901), pp. 40-41.

Image Source: Detail from cover of the Harvard Class Album 1946.

Categories
Chicago Economists Harvard

Harvard. Course Transcript of economics Ph.D. alumnus (1922), Jacob Viner

 

Besides the collection and careful transcription of historical course syllabi and examination questions from leading centers of economics education in the United States, Economics in the Rear-view Mirror also shares information on the structure of undergraduate and graduate economics programs as well as the granular detail found in the transcripts of individual students. 

Recently I posted the Harvard graduate transcript of Edward Chamberlin. Today’s post provides us the Harvard course record of that economist’s economist, Jacob Viner, later of Chicago and Princeton fame.

__________________________

THE GRADUATE SCHOOL OF ARTS AND SCIENCES
Record of Jacob Viner

Years: 1914-15, 1915-16

 

[Previous] Degrees received.

A.B. McGill 1914

First Registration: 28 Sept. 1914

1914-15

Grades

First Year Course

Half-Course

Economics 11

A

Economics 12

A-

Economics 17

A

Economics 33 (full)

A

Economics 34

B+

German A

B+

Division: History, Government, & Economics
Scholarship, Fellowship: University
Assistantship:
Austin Teaching Fellowship:
Instructorship:
Proctorship:
Degree attained at close of year: A.M.

 

1915-16

Grades

Second Year Course

Half-Course

Economics 2a1

A-

Economics 2b2

abs.

Economics 81

A

Economics 14

“excused”

Economics 18a2

cr. for[…]

Economics 31

“exc.”

Philosophy 182

abs.

Philosophy 25a1

A-

Division:
Scholarship, Fellowship: Henry Lee Memorial
Assistantship:
Austin Teaching Fellowship:
Instructorship:
Proctorship:
Degree attained at close of year:  Ph.D. 1922 (Feb.)

Source: Harvard University Archives. Graduate School of Arts and Sciences. Record Cards of Students, 1895-1930, Sun—Walls (UAV 161.2722.5). File I, Box 14, Record Card of Jacob Viner.

__________________________

Courses Names and Professors

1914-15

Economics 11. Economic Theory. Professor Taussig.

Economics 121. (half course) Scope and Methods of Economic Investigation. Professor Carver.

Economics 17. Economic Theory: Value and Related Problems. Assistant Professor B.M. Anderson, Jr.

Economics 33. International Trade and Tariff Problems in the United States. Professor Taussig

Economics 34. Problems of Labor. Professor Ripley.

German A. Elementary Course (prescribed for students who cannot show that they have a satisfactory knowledge of Elementary German)

1915-16

Economics 2a1. European Industry and Commerce in the Nineteenth Century. Professor Gay, assisted by Mr. A.H. Cole and Mr. Ryder.

Economics 2b2. Economic and Financial History of the United States. Professor Gay, assisted by Mr. A.H. Cole and Mr. Ryder.

Economics 81. Principles of Sociology. Professor Carver, assisted by Mr. Bovingdon.

Economics 14. History and Literature of Economics to the year 1848. Professor Bullock.

Economics 18a2. Analytical Sociology. Asst. Professor Anderson.

Economics 31. Public Finance. Professor Bullock.

Philosophy 182. Present Philosophical Tendencies. A brief survey of contemporary Materialism, Pragmatism, Idealism, and Realism.

Philosophy 25a1. Theory of Value. Professor R.B. Perry.

Sources: Harvard University, Faculty of Arts and Sciences. Course of instruction. 1879-2009; Harvard University. Report of the President of Harvard College, 1826-1995.

__________________________

Ph.D. in Economics Awarded 1922

Jacob Viner, A.B. (McGill Univ.) 1914, A.M. (Harvard Univ.) 1915.

Subject, Economics. Special Field, International Trade. Thesis, “The Canadian Balance of International Indebtedness, 1900-1913.”
Assistant Professor of Political Economy, University of Chicago.

Source: Harvard University. Report of the President of Harvard College, 1921-1922, p. 65.

Image Source: Jacob Viner (pipe smoker in the center) playing cards with Messrs. Grabo, Prescott, and Ralph Sanger (mathematician).  University of Chicago Photographic Archive apf1-08487, Special Collections Research Center, University of Chicago Library.

Categories
Harvard Seminar Speakers

Harvard. Economics Seminary. Speakers and Topics, 1914-1915

 

 

 

The economics seminary at Harvard met fourteen times over the course of the 1914-15 academic year.  

An early sighting of Jacob Viner: R. L. Wolf [Robert Leopold Wolf, summa cum laude in Economics, A.B. Harvard 1915] and J. Viner spoke at the Economic Seminary on “The Theory of the Equilibrium of Supply and Demand,” March 29, 1915.

Earlier posts with information on the Seminary of Economics at Harvard:

Seminary of Economics 1897-1898.

Seminary of Economics 1891/92-1907/08.

Seminary of Economics 1913/14.

Request by Radcliffe Women to attend the Seminary of Economics, 1926.

Seminary of Economics 1929-1932.

_______________________

Monday, October 5, 1914

Seminary of Economics. “Studies in Spanish Archives, with Special Reference to the History of the Sheep Owners’ Gild or Mesta.” Mr. Julius Klein [Ph.D. 1915]. Upper Dane, 4.30 p.m.

Source: Harvard University Calendar, Vol. X, No. 2, October 3, 1914.

 

Monday, October 19, 1914

Seminary of Economics. “Combinations in the Book Trade and the Regulation of Retail Prices.” Mr. H. R. Tosdal [Ph.D. 1915]. Upper Dane, 4.30 p.m.

Source: Harvard University Calendar, Vol. X, No. 4, October 17, 1914.

 

Monday, November 2, 1914

Seminary of Economics. “The Contest in Congress between Employers and Trade Unionists.” Mr. P. G. Wright. Upper Dane, 4.30 p.m.

Source: Harvard University Gazette, Vol. X, No. 6, October 31, 1914.

 

Monday, November 23, 1914

Seminary of Economics. “Cotton Manufacturing in Japan.” Mr. R. J. Ray. Upper Dane, 4.30 p.m.

Source: Harvard University Gazette, Vol. X, No. 9, November 21, 1914.

 

Monday, December 7, 1914

Seminary of Economics. “The Tin Plate Industry in Wales and in the United States.” Mr. D. E. Dunbar. Upper Dane, 4.30 p.m.

Source: Harvard University Gazette, Vol. X, No. 11, December 5, 1914.

 

Monday, January 11, 1915

Seminary of Economics. “The Meeting of the American Economic Association.” Professor Carver and Dr. J. S. Davis [Ph.D. 1913]. Upper Dane, 4.30 p.m.

Source: Harvard University Gazette, Vol. X, No. 16, January 9, 1915.

 

Monday, January 25, 1915

Seminary of Economics. “The Development and Organization of the Grain Trade in Canada.” Mr. W. C. Clark. Upper Dane, 4.30 p.m.

Source: Harvard University Gazette, Vol. X, No. 18, January 23, 1915.

 

Monday, February 15, 1915 

Seminary of Economics. “Modern Methods of Real Estate Assessment.” Mr. Alfred D. Bernard, of Baltimore, Md. Upper Dane, 4.30 p.m.

Source: Harvard University Gazette, Vol. X, No. 21, February 13, 1915.

 

Monday, March 1, 1915

Seminary of Economics. “State Board of Conciliation and Arbitration in Massachusetts.” Mr. L. A. Rufener [Ph.D. 1915]. Upper Dane, 4.30 p.m.

Source: Harvard University Gazette, Vol. X, No. 23, February 27, 1915.

 

Monday, March 15, 1915

Seminary of Economics. “The Struggle in the Colorado Coal Mines.” Mr. J. H. Libby. Upper Dane, 4.30 p.m.

Source: Harvard University Gazette, Vol. X, No. 25, March 13, 1915.

 

Monday, March 29, 1915

Seminary of Economics. “The Theory of the Equilibrium of Supply and Demand.” Messrs. R. L. Wolf and J. Viner [Ph.D. 1922]. Upper Dane, 4.30 p.m.

Source: Harvard University Gazette, Vol. X, No. 27, March 27, 1915.

 

Monday, April 12, 1915

Seminary of Economics. “Some Aspects of the Federal Valuation of Railways.” Mr. H. B. Vanderblue [Ph.D. 1915]. Upper Dane, 4.30 p.m.

Source: Harvard University Gazette, Vol. X, No. 29, April 10, 1915.

 

Monday, May 3, 1915

Seminary of Economics. “The Boston and Maine Reorganization.” Professor Ripley. Upper Dane, 4.30 p.m.

Source: Harvard University Gazette, Vol. X, No. 32, May 1, 1915.

 

Monday, May 17, 1915

Seminary of Economics. “The German Steel Kartell.” Mr. H. R. Tosdal [Ph.D. 1915]. Upper Dane, 4.30 p.m.

Source: Harvard University Gazette, Vol. X, No. 34, May 15, 1915.

 

Image Source.  Harvard Square September 23, 1915. “These businesses have weathered decades of change in Harvard Square,” posted at Boston.com.

 

Categories
Bibliography Harvard Socialism Suggested Reading

Harvard. Short Bibliography of the Economics of Socialism for “Serious-minded Students”, Carver, 1910

 

In 1910 Harvard published 43 short bibliographies covering “Social Ethics and Allied Subjects”, about half of which were dedicated to particular topics in economics and economic sociology. The project was coordinated by Plummer Professor of Christian Morals, Francis G. Peabody.

The Economics of Socialism  is one such “allied subject” covered in the bibliography provided by Professor Thomas Nixon Carver, and transcribed below along with links to digital copies of the items found at archive.org, hathitrust.org, as well as at other on-line archives.

Previously posted bibliographies from “Social Ethics and Allied Subjects”:

Economic Theory by Professor Frank Taussig

Taxation by Professor Charles J. Bullock

Trade Unionism by Professor William Z. Ripley

Social Insurance by Dr. Robert Franz Foerster

_____________________________

From the Prefatory Note:

The present list represents an attempt to make this connection between the teaching of the University and a need of the modern world. Each compiler has had in mind, not a superficial reader, nor yet a learned scholar, but an intelligent and serious-minded student, who is willing to read substantial literature if it be commended to him as worth his while and is neither too voluminous nor too inaccessible. To such an inquirer each editor makes suggestions concerning the contents, spirit or doctrine of a book, not attempting a complete description or a final judgment, but as though answering the preliminary question of a student, “What kind of book is this?” The plan thus depends for its usefulness on the competency of the editors concerned, and each editor assumes responsibility for the section to which his name is prefixed.

Source: Prefatory Note by Francis G. Peabody. A Guide to Reading in Social Ethics and Allied Subjects, Lists of Books and Articles Selected and Described for the Use of General Readers. Cambridge, Mass.: Harvard University, 1910, p. vi.

_____________________________

IV.5. THE ECONOMICS OF SOCIALISM
THOMAS NIXON CARVER

I. UTOPIAS

Plato. The republic.

A dialogue on justice, in which the philosopher pictures an ideal state.

 

More, Sir Thomas. Utopia, 1516.

A description of an ideal commonwealth, supposed to have been discovered on the coast of South America by one of the followers of Americus Vespucius.

 

Bacon, Sir Francis. New Atlantis, 1629.

A fragment.

 

Campanella, Tommaso. The city of the sun, 1637.

A highly idealistic picture, sufficiently divorced from all appearances of reality to render it harmless.

 

Cabet, Étienne. Voyage en Icarie, 1840.

Of special interest to Americans because the author led a group of colonists to the United States and established there a communistic society, first at Nauvoo, Ill., and later at Icaria, near Corning, Ia.

 

Gronlund, Laurence. A coöperative commonwealth; an exposition of modern socialism. Fourth edition, London: Swan, Sonnenschein & Co., 1892, pp. 265. [First edition, 1884.]

The first of a large crop of recent utopian works.

 

Bellamy, Edward. Looking backward, 2000-1887. Boston: Houghton, Mifflin Company, 1888, pp. 470.

The most widely read in America of all the utopian works.

 

Morris, William. News from nowhere, or an epoch of rest; being some chapters from a utopian romance. London: Reeves & Turner, 1890, pp. 238.

Probably the most hopelessly idealistic of all such works.

 

Wells, Herbert George. A modern utopia. New York: Charles Scribner’s Sons, 1907, pp. xi, 393.

Probably the only utopian work since Plato’s “Republic” which frankly recognizes the population problem and tries to deal with it.

 

II. COMMUNISTIC EXPERIMENTS

Noyes, John H. History of American socialisms. Philadelphia: J. B. Lippincott & Co., 1870, pp. vi, 678.

The author was the founder of the Oneida community. He had put into his hands for editing and publication the manuscript of A. J. MacDonald, who had made a personal investigation of every communistic society then known to exist on American soil.

 

Nordhoff, Charles. The communistic societies of the United States from personal visit and observation; including detailed accounts of the Economists, Zoarites, Shakers, the Amana, Oneida, Bethel Aurora, Icarian and other existing societies, their religious creeds, social practices, numbers, industries and present conditions. New York: Harper & Brothers, 1875, pp. 439.

 

Hinds, William A. American communities. Revised edition. Chicago: Charles H. Kerr & Co., 1908, pp. 562.

The latest and most authentic account of all the known communistic societies in America.

 

Codman, John T. History of the Brook Farm; historic and personal memoirs. Boston: Arena Publishing Company, 1894, pp. viii, 335.

 

Shaw, Albert. Icaria. New York: G. P. Putnam’s Sons, 1884, pp. ix, 219.

Written before the break-up of the Icarian community, from personal investigation and inspection.

 

Landis, George B. The society of the Separatist of Zoar, annual report of the American Historical Association, 1898. Washington: Government Printing Office, 1899, pp. 163-221.

Written just before the disintegration of the Zoar society, from personal investigation and observation.

 

III. HISTORY OF SOCIALISTIC DOCTRINES

Ely, Richard T. French and German socialism. New York: Harper & Brothers, 1883, pp. 274.

The most readable account in English of the development of socialistic thought in continental Europe since the French revolution.

 

Rae, John. Contemporary socialism. Third enlarged edition. New York: Charles Scribner’s Sons, 1901, pp. 568.

This work brings the subject down to a later period than does Ely’s account. It is also a more voluminous treatment.

 

Peixotto, Jessica. The French revolution and modern French socialism. New York: T. Y. Crowell & Co., 1901, pp. XV, 409.

Perhaps the most discriminating comparison of the two schools of socialism in France, where the dominant school would scarcely be recognized as socialistic by American and German socialists.

 

Hillquit, Morris. History of socialism in the United States. New York: Funk & Wagnalls Co., 1903, pp. 371.

An exceedingly laudatory account, but instructive nevertheless.

 

Guthrie, William B. Socialism before the French revolution. New York: The Macmillan Company, 1907, pp. xviii, 339.

A review of socialistic thought from Thomas More to the radicals of the French revolution.

 

Stoddart, Jane T. The new socialism. New York: George H. Doran Company, 1910, pp. 271.

Rather discursive, but gives a good idea of the present tendency of socialistic thought.

 

IV. IN ADVOCACY OF SOCIALISM

Laveleye, Émile De. The socialism of to-day. Translated by Goddard H. Orpen. London: Field & Iver (1884), pp. viii, 331.

Includes under socialism a great deal which the Marxian socialist would reject.

 

Marx, Karl. Capital, a critical analysis of capitalist production. Translated by Samuel Moore and Edward Aveling. London: Swan, Sonnenschein & Co., 1889, pp. xxxi, 816.

The “bible of socialism.”

 

Marx, Karl, and Engels, Frederick. The communist manifesto. New York: Socialist Co-operative Publishing Association, 1901, pp. 46.

The beginning of the present type of socialist propaganda.

 

Shaw, G. Bernard, editor. Fabian essays in socialism. London: Walter Scott (1890), pp. 233.

A series of essays by such writers as G. Bernard Shaw, Annie Besant, Graham Wallas and others.

 

Engels, Frederick. Socialism, utopian and scientific. Translated by Edward Aveling. New York: Charles Scribner’s Sons, 1892, pp. xxxix, 117.

By scientific socialism is meant the socialism of Karl Marx and his followers.

 

Bernstein, Edward. Ferdinand Lassalle. Translated by Eleanor Marx Aveling. New York: Charles Scribner’s Sons, 1893, pp. xiv, 192.

The author is the leader of the “higher critics” of the socialist school in Germany, which rejects much of the Marxian theory, while adhering to the social democratic program.

 

Bliss, W. D. P. A handbook of socialism. New York: Charles Scribner’s Sons, 1895, pp. viii, 291.

A collection of information about socialism. Apparently intended as a “campaign book” for socialist propagandists.

 

Hyndman, Henry M. The economics of socialism. Second edition. London, 1896, pp. 257.

An attempt to reconstruct the economic basis of socialism. The author’s economic theories are erroneous, but they illustrate very well the kind of reasoning upon which socialists base their claims.

 

Vandervelde, Émile. Collectivism and industrial evolution. Chicago: Charles H. Kerr & Co., 1904, pp. 199.

An excellent presentation, by a socialist of the more rational type, of the general theory of international socialism.

 

Spargo, John. Socialism. New York: The Macmillan Company, 1906, pp. xvi, 257.

Probably the most authoritative statement, in popular form, of the immediate aims of American socialism.

 

MacKaye, James. The economy of happiness. Boston: Little, Brown & Co., 1906, pp. xv, 533.

Probably the only socialistic work since Marx’ “Capital” which seriously tries to lay the foundations of socialism on the recognized principles of economics. As Marx tried to build on the economics of Ricardo, Mackaye tries to build on the economics of the modern school.

 

MacDonald, J. Ramsay. Socialism and government. London: T. C. and E. C. Jack, 1907, pp. vi, 107. [1909 Socialist Library: volume VIII(1) and volume VIII(2)]

Probably the best presentation of the actual working theory of Fabian or English socialism.

 

Wells, Herbert George New worlds for old. New York: The Macmillan Company, 1908, pp. vii, 333.

A daring and ingenious form of propagandism.

 

V. EXPOSITORY AND CRITICAL

Schäffle, Albert. The quintessence of socialism. Translated under supervision of Bernard Bosanquet. New: York: Charles Scribner’s Sons, 1902, pp. viii, 127.

Perhaps the most thorough-going criticism to be found, but not easy to read.

 

Schäffle, Albert. The impossibility of social democracy. New York: Charles Scribner’s Sons, 1892, pp. XX, 419.

This is a supplement to the “Quintessence of socialism.”

 

Ely, Richard T. Socialism: an examination of its nature, strength and weakness. New York: T. Y. Crowell & Co., 1894, pp. xiii, 449.

An eminently fair and sympathetic statement of the pros and cons.

 

Böhm-Bawerk, Eugen von. Karl Marx and the close of his system. Translated by H. M. Macdonald. New York: The Macmillan Company, 1898, pp. 221.

Shows very clearly that Marx built on an antiquated system of economics.

 

Gonner, Edward C. The socialist philosophy of Rodbertus. London: Macmillan & Co., 1899, pp. 234.

A sympathetic study, contrasting Rodbertus with Marx, to the advantage of the former.

 

Le Rossignol, James E. Orthodox socialism: a criticism. New York: T. Y. Crowell & Co., 1907, pp. vii, 147.

By “orthodox” socialism is meant the socialism of Karl Marx. The various tenets of the socialist creed are examined critically.

 

Source: Teachers in Harvard University, A Guide to Reading in Social Ethics and Allied Subjects, Lists of Books and Articles Selected and Described for the Use of General Readers. Cambridge, Mass.: Harvard University, 1910, pp. 167-173.

Image Source: Thomas Nixon Carver in the Harvard Class Album 1915.

Categories
Economists Gender Harvard

Radcliffe. Economics Ph.D. alumna, Elizabeth Boody, 1934

 

Joseph Schumpeter’s third wife, Romaine Elizabeth Firuski née Boody (1898-1953), was the first Radcliffe woman to be awarded the distinction of receiving a summa cum laude A.B. in economics. This post provides a few items from her undergraduate years as well as a brief biography that the Find-A-Grave website clearly copied from somewhere else, but which for our purpose here is still a useful summary. The wedding announcement “Mrs. E.B. Firuski Wed to Educator” from the New York Times (August 17, 1937) provides a wonderful detail regarding the location of the wedding luncheon–the Viennese Roof Garden of the St. Regis in Manhattan.

For much more detail about Elizabeth Boody Schumpeter’s life, career, and her personal and professional partnership with Joseph Schumpeter, see:

Robert Loring Allen, Opening Doors: The Life and Work of Joseph Schumpeter. Volume 2: America. London and New York: Routledge, 1991.

Richard A. Lobdell, “Elizabeth Boody Schumpeter (1898-1953)” in A Biographical Dictionary of Women Economists, Edited by Robert W. Dimand, Mary Ann Dimand, and Evelyn L. Forget. London: Edward Elgar Publishing, 2000, pp. 382-385.

Richard Swedberg, Joseph A. Schumpeter: His Life and Work. Polity Press, 1991.

Elizabeth Boody received her Ph.D. in economics from Radcliffe in 1934. Her doctoral dissertation had the title “Trade Statistics and Cycles in England, 1697-1825”.

___________________________

Radcliffe College Yearbook, 1920

Source: Elizabeth Boody’s senior picture from the Radcliffe Yearbook 1920, p. 36

___________________________

Brief biography from the Find-a-Grave Website

Elizabeth Boody Schumpeter was an economist and expert on East Asia.

Born Romaine Elizabeth Boody on 16 August 1898 in Lawrence, Massachusetts, she was the daughter of Maurice and Hulda (Hokansen) Boody. She lived there with her family until she enrolled at Radcliffe College in the Fall of 1916.

At Radcliffe, Boody majored in economics, pursuing a special interest in labour problems. In the spring of 1920, she was awarded the college’s first summa cum laude AB degree in economics. After graduation, Boody worked as an assistant labour manager for a clothing firm in Rochester, New York. She returned to Radcliffe for graduate studies in economics, including coursework in statistics as well as economics, reflecting the field’s increasing interest in quantitative data and statistical techniques. Boody published her first scholarly article in 1924 in the Review of Economic Statistics, eventually becoming the first woman to serve as a contributing editor of that journal. She earned an M.A. in 1925 and joined the Harvard University Committee on Economic Research, where she was particularly interested in the statistical analysis of time series data and their use in forecasting business cycles. Resuming doctoral studies at Radcliffe, Boody spent 1926 and 1927 collecting English trade statistics for her thesis in London, where she was strongly influenced by Harold Laski and others at the London School of Economics.

Boody was appointed an Assistant Professor of Economics at Radcliffe. She also taught at Vassar (1927-1928) and at Wheaton College (1938-1939, 1948-1949). As a lecturer and author of articles on East Asian economics and politics, she advocated a “moderate isolationist” policy in the Pacific during the years preceding World War II. She was an assistant editor of the Quarterly Journal of Economics.

Boody completed her Ph.D. in 1934. From 1935 to 1940 she worked for the Bureau of International Research at Harvard University. There she directed two studies: one of English trade during the 18th century, and one on the industrialization of Japan and Manchukuo. These resulted in the publication of two books, one of them posthumous: The Industrialization of Japan and Manchukuo (1940) and English Overseas Trade Statistics, 1697-1808 (1960).

In 1937 she married fellow Harvard economist Joseph Alois Schumpeter. He died 08 January 1950 at their residence in the hamlet of Taconic, Town of Salisbury, Litchfield County, CT, where she ran a small nursery. She edited their posthumously published magnum opus, History of Economic Analysis (1954), based on his research.

Elizabeth Boody Schumpeter died of cancer 17 July 1953.

Her personal and professional papers, dating from 1938-1953, are archived at the Arthur and Elizabeth Schlesinger Library on the History of Women in America, Radcliffe Institute, Harvard University, Cambridge, Massachusetts.

___________________________

THE THIRD DIVISION

Sarah Wambaugh, A.B. 1902, A.M. 1917
Romaine Elizabeth Boody, A.B. 1920

Sarah Wambaugh, author of “A Monograph on Plebiscites” and temporary member of the Administration Commission and Minority Section of the Secretariat of the League of Nations, is now an instructor in Political Science at Wellesley College. Romaine Elizabeth Boody graduated summa cum laude in Economics, and became Assistant Employment Manager for the Hickey-Freeman Company of Rochester, New York.

[High likely that Elizabeth Boody is one of the Radcliffe women in the picture below.]

A VISITOR in Cambridge having supper at the Cock Horse, once the home of Longfellow’s “Village Blacksmith,” may occasionally encounter a group of girls in deep discussion. They may be eagerly arguing some point with a man, whom one instantly labels a Harvard professor. The visitor is probably privileged to gaze upon an evening meeting of the Third Division Club of Radcliffe College. The issue may be the League of Nations, the tariff, a decision of the disarmament conference, or any other topic of the day.

The Third Division from which the Club takes its name includes the Departments of History, Government, and Economics. Students concentrating in these departments formed the club some three years ago with a double purpose — to increase the pleasant social intercourse of students and professors interested in the division and to prepare members to pass their final General Examination. When this examination was uppermost in mind, the Club was often unofficially known as the “Third Degree Club.”

Both to Harvard and to Radcliffe large numbers of students have always been drawn from far and wide by the authority and record for public service of the men who give instruction in these departments. But at Radcliffe, interest in these courses has increased greatly during the last few years, until in 1920 approximately one fourth of the Senior class chose this field of concentration. This impetus is traceable in part to the war and to the larger place women are occupying in industrial and social life, but especially to the stimulus of the chance to work under the guidance of men whose names are always in the public print, whose opinions have been anxiously sought at every juncture of the Great War and of the readjustment period.

Regardless of the actual quality of the instruction, is it not human nature to listen the more eagerly to the well-known expert who may come to class occasionally directly from the train from Washington where he has been acting as adviser to a congressional committee? The privilege of hearing and questioning a Thomas Nixon Carver robs the name “sociology” of any impractical flavor it may have had in pre-college days. The labor situation seems to require immediate attention when a Ripley stands ready to interpret it. The newspaper-reading undergraduate who finds Radcliffe her natural habitat is pulled with equal urgency to International Law with George Grafton Wilson, and Municipal Government with William Bennett Munro.

When making up the courses of study for the year it is evident that the fare provided by the Third Division is tantalizing to say the least. How hard it is to choose. How can failure to study under Albert Bushnell Hart, Professor Holcombe, or Professor Day be explained to parents, old teachers, or the neighbors at home? Will one regret the rest of one’s days the omission of Professor Taussig’s course? Most likely. Certain alluring pages in the catalogue must be hurried over. The world seems nothing but one renunciation after another.

In addition to Harvard instruction, Radcliffe students of History, Government, and Economics have the use of the great Harvard Library. They have access to the Boston Public Library and its splendid Americana, to the Boston Athenaeum, famous for its Washingtonia, the Massachusetts State Library, strong on foreign law, and the Library of the Massachusetts Historical Society, rich in local history and manuscript material.

These departments were the first to adopt the tutorial system and the general final examination. Useful as the new plan has proved in other departments, it is especially suited to the study of these subjects. In a literal sense these are living subjects, changing their aspect with each day’s news — news which cannot be correctly interpreted by isolated study but only by discussion. The wide reading necessary must be judiciously assimilated in order to develop the student’s appreciation and critical faculties. This can be done only with the help of some one who had already mastered the subject.

Under the new plan tutors guide and assist the students in preparing for the final examination, meeting those in their charge individually every week. The tutor is in no sense a coach, rather a friendly counselor whose aid is an enormous encouragement to the student in learning how to learn.

It would be interesting to know what these women concentrating in Division Three do after leaving college. After discussing the problems of our present political and industrial structure in the Liberal Club, the Debating Club, and the Third Division Club, do they ever apply their conclusions in practical work? After studying under men of ripe scholarship and wisdom, are they better qualified to take upon themselves the duties of citizenship? These questions are best answered by telling of the work of a few Radcliffe women.

The courses in International Law at Radcliffe have attracted a considerable number of those holding fellowships in the subject from the Carnegie Endowment for International Peace. Two of these graduate students, Bernice V. Brown and Eleanor W. Allen, have subsequently held the Commission for Relief in Belgium Fellowship which means a year’s study in Brussels. A third, Alice Holden, is this year a member of the Department of Government at Smith College, and is giving the course in International Law at that institution.

Many students of economics are engaged in various forms of educational and service work in factories and other industrial establishments, and in administering philanthropies. Elizabeth Brandeis, 1918, is secretary of the Minimum Wage Board of the District of Columbia. Nathalie Matthews, 1907, is the Director of the Industrial Division of the Children’s Bureau at Washington.

The strength of the Third Division lies not alone in the unrivaled quality of the instruction and the stimulus of being in touch with the tide of current history, but also in the type of student it brings to Radcliffe.

SourceWhat We Found at Radcliffe. Boston, McGrath-Sherrill Press, ca. 1921, pp. 7-10.

___________________________

Wedding Announcement

Mrs. E.B. Firuski Wed to Educator

Radcliffe College Research Fellow Married here to Joseph A. Schumpeter

Mrs. Elizabeth Boody Firuski of Windy Hill, Taconic, Conn., was married yesterday at noon to Dr. Joseph A. Schumpeter of Cambridge, Mass., Professor of Economics at Harvard University, in the Community Church of New York by the associate minister, the Rev. Leon Rosser Land.

The ceremony was followed by a luncheon in the Viennese Roof Garden of the St. Regis.

The bride, formerly Assistant Professor of Economics at Vassar College is a research fellow at Radcliffe College, working under the auspices of the Bureau of international Research of Harvard University. Her marriage [1929] to Maurice Firuski was terminated by divorce in Reno in 1933.

Dr. Schumpeter, a widower, was born in Austria, where he was Finance Minister in 1919. He formerly was a professor at the University of Bonn.

Dr. and Mrs. Schumpeter will make their home at Windy Hill until the reopening of the Fall session at Harvard.

Source: The New York Times, August 17, 1937, p. 22.

 

Image Sources: Elizabeth Boody’s senior picture from the Radcliffe Yearbook 1920, p. 36; Portrait of Elizabeth Boody Schumpeter, November 18, 1941. Harvard University Archives.

 

 

Categories
Economists Gender Harvard NBER Radcliffe Smith Vassar

Radcliffe.Economics Ph.D. Alumna, Dorothy Carolin Bacon, 1928

 

This post began after I noticed that it has been some time since I posted biographical and career information for a economics Ph.D. alumna. I figured it would be good to search for a candidate that Economics in the Rear-view Mirror has already caught in an earlier archival trawling expedition but for whom the details of post-doc life had not been added. Dorothy Carolin Bacon was awarded her Radcliffe economics Ph.D. in 1928 and the following item was what I had to start with.

Dorothy Carolin Bacon.

General Examination in Economics, Thursday, May 26, 1927.
Committee: Professors Persons (chairman), Carver, Crum, Gay and Holcombe.
Academic History: Simmons College, 1918-19; Radcliffe College, 1919-22, 1923-24, 1926-. A.B., Radcliffe, 1922; A.M., ibid., 1924. Assistant in Economics, Vassar College, 1924-25. Instructor in Economics, ibid., 1925-26.
General Subjects: 1. Economic Theory. 2. Sociology. 3. History of Political Theory. 4. Statistics. 5. Economic History. 6., Money, Banking and Crises.
Special Subject: Money, Banking and Crises.
Thesis Subject: A Study of the Dispersion of Wholesale Commodity Prices, 1890-1896.  (With Professor Persons.)

Source: Harvard University Archives. Harvard University, Examinations for the Ph.D. (HUC 7000.70), Folder “Examinations for the Ph.D., 1926-1927”.

________________________

One of the items that came up after searching for a Google search was an advertisement for her handwritten Radcliffe student journal notes from her Physics course in 1922. Besides being surprised to see a list price of $750.00 for this notebook, I was intrigued by the relatively detailed information provided about Dorothy Bacon. While everything about the text struck me as fully plausible, I thought it worth some due diligence to confirm what I could from the bookseller’s bio-blurb. I have added links wherever possible to sources that confirm the details below. It would appear that information from the above item in Economics in the Rear-view Mirror as well as from the Dzuback chapters in Madden and Dimand (eds.)  and Margaret A. Nash (ed.) have provided some (or even much) of what was included in the D. Anthem advertisement that follows.

The section on Smith College in Mary Ann Dzuback’s chapter “Women economists in the academy: struggles and strategies, 1900-1940” in the Routledge Handbook of the History of Women’s Economic Thought, Kirsten Madden and Robert W. Dimand (eds.) provides information on Dorothy Bacon from the faculty files of the Smith College Archives [Office of President William Allan Neilson Files, Box 364, Folder 34]:

Bacon came to Smith a year before finishing her Ph.D. at Radcliffe in 1928. She took research and service sabbatical leaves to work for the Works Progress Administration and the Federal Deposit Insurance Corporation in the 1930s. She focused her research on money flows during the 1930s, cost price problems, and the development of federal level credit institutions. By the 1940s, she was working with the federal Office of Price Administration. By the 1950s, she was consulting with the Brookings Institution, had been a research associate of the National Bureau of Economic Research, and received grants from the SSRC. She published a monograph on the recent economic history of five towns around Northampton, Massachusetts, in the late 1930s, and was completing a book on the development of Philippine credit institutions by 1970.

________________________

Another paragraph by Mary Ann Dzuback

From: Mary Ann Dzuback. Chapter 7. Research at Women’s Colleges, 1890-1940. Women’s Higher Education in the United States (Historical Studies in Education), edited by Margaret A. Nash. Palgrave Macmillan, 2018.

Dorothy Bacon (1927–54) arrived in 1928, eventually taking an endowed chair. She investigated the flows of currency during the Depression, cost price problems, and the growth of credit institutions, and was in great demand by private research agencies and the federal government. A sometime consultant with the National Bureau of Economic Research, in the 1920s and 1930s she worked with a range of government and research agencies. She was awarded grants by the Social Science Research Council and published regularly. Bacon’s record of research and service, and her sabbaticals, suggest that women social science scholars at Smith were encouraged to use their research to inform policy at the federal and international levels.

________________________

From: Advertisement for “Economist Dorothy Bacon’s 1922 Physics 2 Journal from Radcliffe College (1922)

Dorothy Carolin Bacon was born in 1902 to George Preston Bacon, a professor of Physics and Dean of both the Tufts Engineering School and the Bromfield-Pearson School, and Hannah Churchill Bacon, a trained nurse. Her sister, Ruth Bacon, also attended Radcliffe College and later became the first female officer of a State Department geographical bureau (Bureau of Eastern Affairs). Bacon attended Simmons College from 1918-19 before transferring to Radcliffe, the former women’s liberal arts college that fully merged with Harvard in 1999. She earned her B.A. (1922), M.A. (1924) and Ph.D (1927) [sic, 1928] there with her dissertation concerning A Study of the Dispersion of Wholesale Commodity Prices, 1890-1896. While at Radcliffe she also worked for the Federal Reserve Board’s Division of Research and Analysis [as a Statistical Clerk starting 1 July 1922earning an annual salary of $1600 before resigning [May 10] in 1923.

She was hired as an assistant professor [sic, “Assistant” is a lower rank than “Assistant Professor”] in economics at Vassar in 1924 [cf. AER, Vol. 14, No. 4 (Dec. 1924), p. 829 “Miss Dorothy C. Bacon is assistant in economics at Vassar College.”], but was recruited by Esther Lowenthal, Dean of the Faculty and chair of the economics department at Smith, to join Smith’s faculty in 1927. At Smith she focused her research on money flows during the 1930s, cost price problems, and the development of credit institutions at the federal level. In 1930, she was one of three research associates selected for the National Bureau of Economic Research where she studied the relation of current stock prices to earnings per share from the twenty corporations comprising the Index of Industrial Stock Prices of the Harvard Economic Service. Her monograph, Recent Economic History of the Five Towns (1937) was published by the Works Progress Administration. In 1942, Bacon left her post at Smith [sic, only temporary leave] to work under Leon Henderson at the Office of Price Administration. It was there that she wrote a study of the scrap metal market in Syracuse, NY. By the 1950s, she was consulting with the Brookings Institution and was publishing her research in the Review of Economic Statistics, the Journal of the American Statistical Association and the National Encyclopedia. She appears to have never married [she wasn’t]  and when she died in 1998 she was buried at Shawsheen Cemetery in Bedford, MA, alongside her parents and sister.

Source: D. Anthem, Bookseller advertisement for “Economist Dorothy Bacon’s 1922 Physics 2 Journal from Radcliffe College (1922) [posted price: $750.00!]

________________________

A.E.A. Biographical Listing, 1969

BACON, Dorothy Carolin, academic; b. Beloit, Wis., 1902; student Simmons Coll., 1918-19; A.B., Radcliffe Coll., 1922, A.M., 1924, Ph.D., 1928. FIELDS 2c, 5e, 4a. Research asso., Nat. Bur. Econ. Research, 1930-31; formerly sr. research asso., Fed. Deposit Ins. Corp.; fed. Dir. Research project, Work Progress Adm., 1935-36; asst. div. economist, food price div., OPA [Office of Price Administration], 1943-47, OPS [Office of Price Stabilization], 1951; Fulbright prof., U. Philippines, 1956-57; mem. Faculty, Smith Coll. Since 1927, prof. since 1938, Robert A. Woods prof. since 1956. ADDRESS Smith Coll., 115 Elm St., Northampton, MA 01060.

Note. Fields: 2c (Economic Development Studies); 5e (General International Economics); 4a (Monetary and Financial Theory and Institutions).

SourceThe American Economic Review, Vol. 59, No. 6, 1969. Handbook of the American Economic Association (January 1970), p. 17.

________________________

Bachelor of Arts, Radcliffe (1922)

With Distinction in Special Subjects
Cum Laude

Dorothy Carolin Bacon [of] Medford. In Mathematics.

Source: Report of the Dean in Annual Report of Radcliffe College for 1920-1923, p. 43.

________________________

Master of Arts, Radcliffe (1924)

Dorothy Carolin Bacon, A.B.

Source: Report of the Dean in Annual Report of Radcliffe College for 1923-1924, p. 31.

________________________

Doctor of Philosophy, Radcliffe (1928)

Dorothy Carolin Bacon A.M.

Subject, Economics. Special Field, Money and Banking. Dissertation, “Maladjustment of Prices with Special Reference to the Wholesale Prices of Commodities in the United States; 1890-1896”

Source:  Report of the Dean in Annual Reports of Radcliffe College for 1927-1928, p. 23.

________________________

Publications of Dorothy C. Bacon

A Monthly Index of Commodity Prices, 1890-1900. Review of Economics and Statistics, Vol. 8, No. 4 (October 1926), pp. 177-83.

The Significance of Fixed-base and Link Relatives in Studies of Price Stability: A Comment on the Behavior of Prices. Journal of the American Statistical Association, Vol. 23 (September 1928), pp. 274-81.

Maladjustment of Prices with Special Reference to the Wholesale Prices of Commodities in the United States, 1890-1896. Ph.D. thesis, Radcliffe College.

Encyclopedia articles in the National Encyclopedia.

Recent Economic History of Five Towns. Northampton, Mass.: Smith College, 1937.

________________________

Vital Dates and Miscellaneous Information

Born:  25 February 1902 in Beloit, Wisconsin.

Last Residence: Niceville, Okaloosa, Florida [Socal Security death index].

Died: 8 November in Meriden, New Haven County, Connecticut. [Apparently visiting: the Connecticut Death Index notes her address 2475 Virginia, Residence Andover, District of Columbia].

________________________

Image Source: Senior year picture of Dorothy Carolin Bacon in the  Radcliffe Year Book 1922, p. 23.

 

 

 

Categories
AEA

American Economic Association. Economic Studies, 1896-1899

 

A few posts ago I put together a list of links to the contents of eleven volumes of monographs published by the American Economic Association from 1886 through 1896.

Those eleven published volumes were briefly followed (1896-1899) by two series of AEA publications, viz.: the bi-monthly Economic Studies, and an extremely short “new series” of larger monographs that would be printed at irregular intervals. In 1900 the American Economic Association reverted to the policy of issuing its monographs, called the “third series” of the publications, at quarterly intervals.

This post provides links to the 1896-1899 intermezzo of AEA publications.

______________________

American Economic Association
ECONOMIC STUDIES.

Price of the Economic Studies $2.50 per volume in paper, $3.00 in cloth. The set of four volumes, in cloth, $10.00.

VOLUME I, 1896
[prices in paper]

No. 1 (Apr., Supplement) Eighth Annual Meeting: Hand-Book and Report. Pp. 178. Price 50 cents.

No. 1 (Apr.). The Theory of Economic Progress, by John B. Clark, Ph.D.; The Relation of Changes in the Volume of the Currency to Prosperity, by Francis A. Walker, LL.D. Pp. 46. Price 50 cents.

No. 2 (Jun.). The Adjustment of Wages to Efficiency. Three papers: Gain Sharing, by Henry R. Towne; The Premium Plan of Paying for Labor, by F.A. Halsey; A Piece-Rate System, by F.W. Taylor. Pp. 83 Price 50 cents.

No. 3 (Aug.). The Populist Movement. By Frank L. McVey, Ph.D. Pp. 81 Price 50 cents.

No. 4 (Oct.). The Present Monetary Situation. An address by Dr. W. Lexis, University of Göttingen translated by Professor John Cummings. Pp. 72. Price 75 cents.

Nos. 5-6 (Dec.). The Street Railway Problem in Cleveland. By W.R. Hopkins. Pp. 94. Price 50 cents.

 

VOLUME II, 1897

No. 1 (Feb., Supplement). Ninth Annual Meeting: Hand-Book and Report. Pp. 162. Price 50 cents.

No. 1 (Feb.). Economics and Jurisprudence. By Henry C. Adams, Ph.D. Pp. 48. Price 50 cents.

No. 2 (Apr.). The Saloon Question in Chicago. By John E. George, Ph.B. Pp. 62. Price 50 cents.

No. 3 (Jun.). The General Property Tax in California. By Carl C. Plehn, Ph.D. Pp. 88. Price 50 cents.

No. 4 (Aug.). Area and Population of U. S. at Eleventh Census. By Walter F. Willcox, Ph.D. Pp. 60. Price 50 cents.

No. 5 (Oct.). A Discourse Concerning the Currencies of the British Plantations in America, etc. By William Douglass. Edited by Charles J. Bullock, Ph.D. Pp. 228. Price 50 cents.

No. 6 (Dec.). Density and Distribution of Population in U.S. at Eleventh Census. By Walter F. Wilcox, Ph.D. Pp. 79.Price 50 cents.

 

VOLUME III, 1898

No. 1 (Feb., Supplement). Tenth Annual Meeting: Hand-Book and Report. Pp. 136. Price 50 cents.

No. 1 (Feb.). Government by Injunction. By William H. Dunbar, A.M., LL.B. Pp. 44. Price 50 cents.

No. 2 (Apr.). Economic Aspects of Railroad Receiverships. By Henry H. Swain, Ph.D. Pp. 118. Price 50 cents.

No. 3 (Jun.). The Ohio Tax Inquisitor Law. By T. N. Carver, Ph.D. Pp. 50. Price 50 cents.

No. 4 (Aug.). The American Federation of Labor. By Morton A. Aldrich, Ph.D. Pp. 54. Price 50 cents.

No. 5 (Oct.). Housing of the Working People in Yonkers. By Ernest Ludlow Bogart, Ph.D. Pp. 82. Price 50 cents.

No. 6 (Dec.). The State Purchase of Railways in Switzerland. By Horace Micheli; translated by John Cummings, Ph.D. Pp. 72. Price 50 cents.

 

VOLUME IV, 1899

No. 1 (Feb.). I. Economics and Politics. By Arthur T. Hadley, A.M.; II. Report on Currency Reform. By F. M. Taylor, F.W. Taussig, J.W. Jenks, Sidney Sherwood, David Kinley; III. Report on the Twelfth Census. By Richmond Mayo-Smith, Walter F. Willcox, Carroll D. Wright, Roland P. Falkner, Davis R. Dewey. Pp.70. Price 50 cents.

No. 2 (Apr.). Eleventh Annual Meeting: Hand-Book and Report. Pp. 126. Price 50 cents.

No. 2 (Apr.). Personal Competition: Its Place in the Social Order and Effect upon Individuals; with some Consideration upon Success. By Charles H. Cooley, Ph.D. Pp. 104. Price 50 cents.

No. 3 (Jun.). Economics as a School Study. By Frederick R. Clow, A.M. Pp. 72. Price 50 cents.

Nos. 4-5 (Aug.-Oct.). The English Income Tax, with Special Reference to Administration and Method of Assessment. By Joseph A. Hill, Ph.D. Pp. 162. Price $1.00.

No. 6. (Dec.) The Effects of Recent Changes in Monetary Standards upon the Distribution of Wealth. By Francis Shanor Kinder, A.M. Pp.91. Price 50 cents.

______________________

NEW SERIES

No. 1 (Dec., 1897). The Cotton Industry. By M. B. Hammond. Pp. 382. (In cloth $2.00.) Price $1.50.

No. 2 (Mar., 1899). Scope and Method of the Twelfth Census. Critical discussion by over twenty statistical experts. Pp. 625. (In cloth $2.50.) Price $2.00.

 

 

Categories
Exam Questions Harvard Suggested Reading Syllabus

Harvard. Core economic theory. Readings and Exams. Carver, 1900/01-1902/03

 

 

For the academic years 1900/01 through 1902/03 the core course in economic theory at Harvard was taught by Thomas Nixon Carver. He was substituting for Frank Taussig, who later wrote that he had been “compelled by ill health to withdraw from teaching” (1901-03). [Chapter IX Economics (1871-1929) by The Development of Harvard University since the Inauguration of President Eliot, 1869-1929 Cambridge: Harvard University Press, 1930. p. 191].  

Schumpeter provided more detail: “We speak of nervous breakdown in such cases, which indeed are more frequent in the academic profession than one would infer from the general conditions of a professor’s life. He [Taussig] took leave and went abroad for two years, relaxing completely and spending one winter at Meran in the Austrian Alps, another on the Italian Riviera, and the summer between (1902) in Switzerland. Catastrophe was thus avoided, and in the fall of 1903 he was able to return to teaching and the editorship of the Quarterly Journal.” [Joseph A. Schumpeter, Chapter 7 “Frank William Taussig (1859-1940)” in Ten Great Economists from Marx to Keynes. p. 206.]

During the first term of 1903/04 Taussig resumed teaching the core economic theory course with Carver teaching the second term.  Beginning 1904/05 Taussig once again taught the course by himself.

___________________________

Economics 2.
Economic Theory in the Nineteenth Century
1900-01

Enrollment
1900-01

For Undergraduates and Graduates:—

[Economics] 2. Asst. Professor Carver.— Economic Theory in the Nineteenth Century.

Total 45: 6 Graduates, 15 Seniors, 16 Juniors, 5 Sophomores, 3 Other.

Source: Harvard University. Annual Reports of the President and Treasurer of Harvard College, 1900-01, p. 64.

 

Reading list [previously posted]  for the first term

 

ECONOMICS 2
[Mid-year examination, 1901]

  1. Define value and explain why one commodity possesses more value in proportion to its bulk than another.
  2. Explain the various uses of the term diminishing returns, and define it as you think it ought to be defined.
  3. In what sense does a law of diminishing returns apply to all the factors of production.
  4. State briefly Böhm-Bawerk’s explanation of the source of interest.
  5. What, if any, is the relation of abstinence to interest.
  6. Would you make any distinction between the source of wages and the factors which determine rates of wages? If so, what? If not, why not?
  7. Discuss the question: Is a demand for commodities a demand for labor?
  8. What is the relation of the standard of living to wages.
  9. Discuss briefly the following questions relating to speculators’ profits. (a) Do speculators as a class make any profits? (b) Are speculators’ profits in any sense earned?
  10. In what sense, if any, does the value of money come under the law of marginal utility?

Source: Harvard University Archives. Harvard University Mid-year Examinations, 1852-1943. Box 4, Bound volume: Examination Papers, Mid-Years, 1900-01.

 

ECONOMICS 2
[Final examination, June 1901]

Discuss the following topics.

  1. The bearing of the marginal utility theory of value upon the questions of wages and interest.
  2. The definitions of capital as given by Taussig and Clark.
  3. Clark’s explanation of the place of distribution within the natural divisions of economics.
  4. Clark’s method of distinguishing between the product of labor and the product of capital.
  5. Clark’s distinction between rent and interest.
  6. Böhm-Bawerk’s theory of the nature of capital.
  7. The origin of capital, according to Böhm-Bawerk and Clark.
  8. The meaning of the word “productive” in the following proposition: “Protection is an attempt to attract labor and capital from the naturally more productive, to the naturally less productive industries.”
  9. The incidence of tariff duties.
  10. The theory of production and the theory of valuation as the two principal departments of economics.

Source: Harvard University Archives. Harvard University, Examination Papers, 1873-1915. Box 5, Bound volume: Examination Papers, 1900-01. Papers Set for Final Examinations in History, Government, Economics, Philosophy, Education, Fine Arts, Architecture, Landscape Design, Music in Harvard College (June, 1901), pp. 23-24.

___________________________

Economics 2.
Economic Theory
1901-02

Enrollment
1901-02

For Undergraduates and Graduates:—

[Economics] 2. Asst. Professor Carver.— Economic Theory.

Total 32: 5 Graduates, 6 Seniors, 17 Juniors, 2 Sophomores, 2 Others.

Source: Harvard University. Annual Reports of the President and Treasurer of Harvard College, 1901-02, p. 77.

ECONOMICS 2.
1901-1902

General Reading. Prescribed.

Marshall. Principles of Economics.
Taussig. Wages and Capital.
Böhm-Bawerk. Positive Theory of Capital.
Clark. The Distribution of Wealth.

References for Collateral Reading. Starred references are prescribed.

I. VALUE.

1. Adam Smith. Wealth of Nations. Book I. Chs. 5, 6, and 7.

2. Ricardo. Pol. Econ. Chs. 1 and 4.

3. Mill.          “        “     Book III. Chs. 1-6.

4.  Cairnes     “        “     Part I.

5.*  Jevons. Theory of Pol. Econ. Chs. 2-4.

6.   Sidgwick. Pol. Econ. Book II. Ch. 2.

7.   Wieser. Natural Value.

8.* Clark. Philosophy of Wealth. Ch. 5

II. DIMINISHING RETURNS.

1.    Senior. Pol. Econ. Pp. 81-86.

2*.  Commons. The Distribution of Wealth. Ch. 3.

III. RENT.

1.   Adam Smith. Wealth of Nation. Book I. Ch. 2. Pts. 1-3.

2.* Ricardo. Pol. Econ. Chs. 2 and 3.

3.   Sidgwick. “     “       Book II. Ch. 7.

4.   Walker.     “     “       Pt. IV. Ch. 2.

5.   Walker. Land and its Rent.

6.  Hyde. The Concept of Price Determining Rent. Jour. Pol. Econ. V.6. p. 368.

7.  Fetter. The Passing of the Old Rent Concept. Q.J.E. Vol. XV. P. 416.

IV. CAPITAL

1.   Adam Smith. Wealth of Nations. Book II.

2.   Senior. Pol. Econ. P. 58-81.

3.   Mill.        “       “       Book I. Ch. 4-6.

4.   Roscher. “      “       Book I. Ch. 1. Secs. 42-45.

5.   Cannan. Production and Distribution. Ch. 4.

6.   Jevons. Theory of Political Economy Ch. 7.

7.  Fisher. What is Capital? Economic Journal. Vol. VI. P. 509.

8.  Fetter. Recent Discussion of the Capital Concept. Q.J.E. Vol. XV. P. 1.

9.* Carver. Clark’s Distribution of Wealth. Q.J.E., Aug. 1901.

V. INTEREST.

1.   Adam Smith. Wealth of Nations. Book I. Ch. 9.

2.   Ricardo. Pol. Econ. Ch. 6.

3.   Sidgwick.  “     “        Book II. Ch. 6.

4*.  Carver. Abstinence and the Theory of Interest. Q.J.E, Vol. VIII. P. 40.

5.    Mixter. Theory of Saver’s Rent. Q.J.E. Vol. XIII. P. 345.

VI. WAGES.

1.   Adam Smith. Wealth of Nations. Book I. Ch. 8.

2*. Ricardo. Pol. Econ. Ch. 5.

3.   Senior.       “       “      Pp. 141-180 and 200-216.

4.   Senior. Lectures. Pp. 1-62.

5.   Mill. Pol. Econ. Book II. Chs. 11, 12, 13, and 14.

6.   Cairnes. Pol. Econ. Part II. Chs. 1 and 2.

7. Sidgwick.  “       “      Book II. Ch. 8.

8.  Walker.     “       “      Part IV. Ch. 5.

9.  Hadley. Economics. Ch. 10.

10*. Carver. Wages and the Theory of Value. Q.J.E. Vol. VIII, P. 377.

VII. PROFITS.

1.   Walker. Pol. Econ. Part IV. Ch. 4.

2.   Hobson. The Law of the Three Rents. Quar. Jour. Econ. Vol. V. P. 263.

3.   Clark. Insurance and Business Profits. Quar. Jour. Econ. Vol. VII. P. 40.

4*.  Hawley, F. B. in Quar. Jour. Econ. Vol. VII. P. 459; Vol. XV. Pp. 75 and 603.

5.    MacVane, in in Quar. Jour. Econ.,  Vol. II. P. 1.

6.   Haynes, in               “     “       “     Vol. IX, P. 409.

Source: Harvard University Archives. HUC 8522.2.1, Box 1 of 10 (Syllabi, course outlines and reading lists in Economics, 1895-2003). Folder: 1901-1902.

 

ECONOMICS 2
[Mid-year examination, 1902]

Discuss the following topics.

  1. The relation of utility to value.
  2. The price of commodities and the price of services.
  3. Various uses of the term “diminishing returns.”
  4. The law of diminishing returns as applied to each of the factors of production.
  5. Prime and supplementary cost: illustrate.
  6. Joint and composite demand and join and composite supply.
  7. Quasi rent.
  8. Real and nominal rent.
  9. Consumer’s rent.
  10. The equilibrium of demand and supply

Source: Harvard University Archives. Harvard University Mid-year Examinations, 1852-1943. Box 6, Bound volume: Examination Papers, Mid-Years, 1901-02.

 

ECONOMICS 2
[Final examination, June 1902]

  1. State some of the different meanings which have been given to the law of diminishing returns, and define the law as you think it ought to be.
  2. Can you apply the law of joint demand to the wages fund questions?
  3. What is meant by an elastic demand and how does it affect monopoly price.
  4. Discuss Clark’s distinction between capital and capital goods.
  5. Under what conditions would there be no rent, and how would these conditions affect the value of products?
  6. Explain Clark’s theory of Economic Causation.
  7. What is the source of interest?
  8. What is the relation of the standard of living to wages?

Source: Harvard University Archives. Harvard University, Examination Papers, 1873-1915. Box 6, Bound volume: Examination Papers, 1902-03. Papers Set for Final Examinations in History, Government, Economics, Philosophy, Education, Fine Arts, Architecture, Landscape Architecture, Music in Harvard College (June, 1902), p. 21.

___________________________

Economics 2.
Economic Theory
1902-03

Enrollment
1902-03

For Undergraduates and Graduates:—

[Economics] 2. Professor Carver.— Economic Theory.

Total 25: 5 Graduates, 8 Seniors, 7 Juniors, 3 Sophomores, 2 Others.

Source: Harvard University. Annual Reports of the President and Treasurer of Harvard College, 1902-03, p. 67.

 

Course Description
1902-03

For Undergraduates and Graduates

[Economics] 2. Economic Theory. Mon., Wed., Fri., at 2.30 Professors Taussig [sic] and Carver.

Course 2 is intended to acquaint the student with some of the later developments of economic thought, and at the same time to train him in the critical consideration of economic principles and the analysis of economic conditions. The exercises are accordingly conducted mainly by the discussion of selected passages from the leading writers: and in this discussion of selected passages from the leading writers; and in this discussion the students are expected to take an active part. Lectures are given at intervals outlining the present condition of economic theory and some of the problems which call for theoretical solution. Theories of value, diminishing returns, rent, wages, interest, profits, the incidence of taxation, the value of money international trade, and monopoly price, will be discussed. Marshall’s Principles of Economics [4th ed., 1898], Böhm-Bawerk’s Positive Theory of Capital [1888; William Smart translation, 1891], Taussig’s Wages and Capital [1896], and Clark’s Distribution of Wealth [1899] will be read and criticized.

Course 2 is open to students who have passed satisfactorily in Course 1.

Source: Harvard University.  Faculty of Arts and Sciences. Division of History and Political Science Comprising the Departments of History and Government and Economics, 1902-03. The University Publications, New Series, No. 55 (June 14, 1902), pp. 40-41.

 

ECONOMICS 2.
1902-1903

General Reading. Prescribed.

Marshall. Principles of Economics.
Taussig. Wages and Capital.
Böhm-Bawerk. Positive Theory of Capital.
Clark. The Distribution of Wealth.

References for Collateral Reading. Starred references are prescribed.

I. VALUE.

1.  Adam Smith. Wealth of Nations. Book I. Chs. 5, 6, and 7.

2.  Ricardo. Pol. Econ. Chs. 1 and 4.

3.   Mill.          “        “     Book III. Chs. 1-6.

4.   Cairnes     “        “     Part I.

5.*  Jevons. Theory of Pol. Econ. Chs. 2-4.

6.   Sidgwick. Pol. Econ. Book II. Ch. 2.

7.   Wieser. Natural Value.

8.* Clark. Philosophy of Wealth. Ch. 5

II. DIMINISHING RETURNS.

1.  Senior. Pol. Econ. Pp. 81-86.

2.  Commons. The Distribution of Wealth. Ch. 3.

3*. Bullock. The Variation of Productive Forces, Q.J.E., August, 1902.

III. RENT.

1.  Adam Smith. Wealth of Nation. Book I. Ch. 2. Pts. 1-3.

2.* Ricardo. Pol. Econ. Chs. 2 and 3.

3.  Sidgwick. “     “       Book II. Ch. 7.

4.  Walker.     “     “       Pt. IV. Ch. 2.

5.  Walker. Land and its Rent.

6.  Hyde. The Concept of Price Determining Rent. Jour. Pol. Econ. V.6. p. 368.

7.  Fetter. The Passing of the Old Rent Concept. Q.J.E. Vol. XV. P. 416.

IV. CAPITAL

1.  Adam Smith. Wealth of Nations. Book II.

2.  Senior. Pol. Econ. P. 58-81.

3.  Mill.        “       “       Book I. Ch. 4-6.

4.  Roscher. “      “       Book I. Ch. 1. Secs. 42-45.

5. Cannan. Production and Distribution. Ch. 4.

6.  Jevons. Theory of Political Economy Ch. 7.

7.  Fisher. What is Capital? Economic Journal. Vol. VI. P. 509.

8.  Fetter. Recent Discussion of the Capital Concept. Q.J.E. Vol. XV. P. 1.

9.* Carver. Clark’s Distribution of Wealth. Q.J.E., Aug. 1901.

V. INTEREST.

1.   Adam Smith. Wealth of Nations. Book I. Ch. 9.

2.   Ricardo. Pol. Econ. Ch. 6.

3.   Sidgwick.  “     “        Book II. Ch. 6.

4*. Carver. Abstinence and the Theory of Interest. Q.J.E, Vol. VIII. P. 40.

5.   Mixter. Theory of Saver’s Rent. Q.J.E. Vol. XIII. P. 345.

VI. WAGES.

1.   Adam Smith. Wealth of Nations. Book I. Ch. 8.

2*. Ricardo. Pol. Econ. Ch. 5.

3.  Senior.       “       “      Pp. 141-180 and 200-216.

4.   Senior. Lectures. Pp. 1-62.

5.   Mill. Pol. Econ. Book II. Chs. 11, 12, 13, and 14.

6.   Cairnes. Pol. Econ. Part II. Chs. 1 and 2.

 7.  Sidgwick.  “       “      Book II. Ch. 8.

8.  Walker.     “       “      Part IV. Ch. 5.

9.  Hadley. Economics. Ch. 10.

10*. Carver. Wages and the Theory of Value. Q.J.E. Vol. VIII, P. 377.

VII. PROFITS.

1.   Walker. Pol. Econ. Part IV. Ch. 4.

2.   Hobson. The Law of the Three Rents. Quar. Jour. Econ. Vol. V. P. 263.

3.   Clark. Insurance and Business Profits. Quar. Jour. Econ. Vol. VII. P. 40.

4*.  Hawley, F. B. in Quar. Jour. Econ. Vol. VII. P. 459; Vol. XV. Pp. 75 and 603.

5.   MacVane, in in Quar. Jour. Econ.,  Vol. II. P. 1.

6.   Haynes, in               “     “       “     Vol. IX, P. 409.

Source: Harvard University Archives. HUC 8522.2.1, Box 1 of 10 (Syllabi, course outlines and reading lists in Economics, 1895-2003). Folder: 1902-1903.

 

ECONOMICS 2
[Mid-year examination, 1903]

Explain and illustrate any twelve of the following subjects.

  1. Marginal utility.
  2. Elasticity of wants.
  3. The law of diminishing returns from land.
  4. The extension of the law of diminishing returns to other factors than land.
  5. The law of economy of organization (Bullock).
  6. The law of varied costs (Bullock).
  7. The cause of rent.
  8. The law of rent.
  9. Quasi rent.
  10. Joint and composite demand.
  11. Joint and composite supply.
  12. Prime and supplementary cost.
  13. The relation of rent to the price of products.
  14. The effect of the shortening of the working day upon the demand for labor.

Source: Harvard University Archives. Harvard University Mid-year Examinations, 1852-1943. Box 6, Bound volume: Examination Papers, Mid-Years, 1902-03.

 

ECONOMICS 2
[Final examination, June 1903]

  1. Explain the principal of marginal utility.
  2. Explain the law of diminishing returns and extend it to other factors than land.
  3. What is the relation of cost to value?
  4. What is the relation of rent to value?
  5. What is the relation of waiting to interest?
  6. What is capital?
  7. What is the relation of capital to wages?
  8. Explain joint and composite demand and joint and composite supply.
  9. Does the home consumer necessarily pay the whole of the tariff duty?
    Give reasons for your answer.
  10. Is the value of money determined in all particulars as the value of any other commodity?

Source: Harvard University Archives. Harvard University, Examination Papers, 1873-1915. Box 6, Bound volume: Examination Papers, 1902-03. Papers Set for Final Examinations in History, Government, Economics, History of Religions, Philosophy, Education, Fine Arts, Architecture, Landscape Architecture, Music in Harvard College (June, 1903), p. 21.

Source Image: Thomas Nixon Carver, Harvard Class Album 1906.

 

Categories
Agricultural Economics Harvard Problem Sets

Harvard. Problem set from agricultural economics. Carver, ca. 1904

 

The problem set transcribed below was found in the Harvard University archives collection of syllabi, course outlines and reading lists in economics, 1895-2003; box 1. It was (mis-)filed in the folder “Economics, 1904-05”.  The problem set is clearly identified as belonging to Economics 23. This semester course, “The Economics of Agriculture, with special reference to American conditions”, was taught by Professor Thomas Nixon Carver, but according to the annual report of the president of Harvard College, the course was not offered in 1904-05 though it was indeed offered during the immediately preceding academic year. I have assumed that the problem set was printed for the second term of the academic year 1903-1904. This is consistent with the library time stamp on the problem set (March 7, 1905), i.e. it cannot have come from later years.

From Carver’s autobiography, Recollections of an Unplanned Life, we know that his textbook, Principles of Rural Economics (1911) was based upon this course. For a long-form course reading list, one can consult the bibliography, pp. xi-xviii, in the textbook.

Previously transcribed and posted artifacts from Carver’s agricultural economics course:

Course enrollment and final exam for 1914-15.

Course syllabus for 1917.

Course examination from 1918.

________________________

Trace of the 1904 problem set found in Carver’s 1911 textbook

Note:  Column (Field A) is Table A p. 180; Column (Field C) is Table B p. 181

Source: Thomas Nixon Carver, Principles of Rural Economics, (1911).

________________________

From Thomas Nixon Carver’s Autobiography.

I have mentioned the three strenuous years 1900 to 1903, and that I served the three following years, 1903-1906, as chairman of the Division of History, Government, and Economics. Before leaving for my sabbatical year abroad in 1906, I had resigned as chairman of the Division. In the fall of 1907 I was back in Cambridge with no administrative responsibilities and ready to settle down to teaching and writing. By this time I had come to be recognized as one of the pioneers in this country in the field of agricultural economics. One of the difficulties in the teaching of that subject was the lack of written material. Textbooks were needed and I began to plan one of my own. Before I got well started Professor Liberty Hyde Bailey of Cornell asked me to write a brief historical sketch of American agriculture for his Cyclopedia of American Agriculture which he was preparing. I under took this, not realizing how much work it would require. The material, such as there was, was widely scattered and there was no guide to indicate where to look for it. However, with much toil and sweat I finished the chapter.

Then came a request for an account of the introduction of various crops and farm animals into this country. That was a still harder job but I finished it in time. I was able, later, to use a part of the material in my book, “Principles of Rural Economics,” which Ginn & Company published in 1911.

This book did a great deal to popularize agricultural economics in this country. Henry C. Taylor’s “Introduction to the Study of Agricultural Economics” had preceded it, but, while an excellent introduction, had not made much of an appeal outside the agricultural colleges. My “Principles” sold well. As I remember it, 40,000 copies were sold the first year, and it was favorably reviewed in a number of journals…

…The course on rural economics appealed to a limited number of students, but continued to be elected by enough to make a fair-sized class…

Source: Thomas Nixon Carver. Recollections of an Unplanned Life, p. 171.

________________________

Course description

[Economics] 23 2hf. The Economics of Agriculture, with special reference to American conditions. Half-course (second half-year). Tu., Th., at 1.30. Professor Carver.

Omitted in 1904-05.

            A study of the relation of agriculture to the whole industrial system, the relative importance of rural and urban economics, the conditions of rural life in different parts of the United States, the forms of land tenure and methods of rent payment, the comparative merits of large and small holdings, the status and wages of farm labor, the influence of farm machinery, farmers’ organizations, the marketing and distribution of farm products, agricultural credit, the economic aspects of public roads, irrigation, forestry, etc., the policy of the government toward agriculture, and the probable future of American agriculture.

The course will be conducted by means of lectures, discussions and reports, with some special investigations of local conditions.

 

Source: Harvard University. Faculty of Arts and Sciences. Division of History and Political Science comprising the Departments of History and Government and Economics, 1904-05. University Publications, New Series, No. 129 (May 16, 1904), p. 47.

________________________

Course enrollment, 1903-04

[Economics] 23 2hf. Professor Carver.—The Economics of Agriculture, with special reference to American conditions.

Total 99: 5 Graduates, 32 Seniors, 28 Juniors, 17 Sophomores, 2 Freshmen, 15 Others.

Source: Harvard University. Report of the President of Harvard College, 1903-04, p. 67.

*  *  *  *  *  *  *  *

Note:  The course was indeed not offered in 1904-05, though course enrollments were reported for Carver’s courses Economic 3 “Principles of Sociology. Theories of Social Progress”; Economics 13 “Methods of Economic Investigation”; Economics 14a “The Distribution of Wealth”; Economics 14b “Methods of Social Reform. Socialism, Communism, the Single Tax, etc.”

Source: Harvard University. Report of the President of Harvard College, 1904-05, pp. 74 ff.

*  *  *  *  *  *  *  *

Course enrollment, 1905-06

[Economics] 23 2hf. Professor Carver.—The Economics of Agriculture, with special reference to American conditions.

Total 42: 4 Graduates, 10 Seniors, 11 Juniors, 14 Sophomores, 1 Freshman, 2 Others.

Source: Harvard University. Report of the President of Harvard College, 1905-06, p. 73.

________________________

Time stamp: “Harvard College Library, MAR 7, 1905”

HARVARD UNIVERSITY
ECONOMICS 23

Amount of corn grown with varying amounts of labor on a given amount of land.

Number of days’ labor of a man and team with the appropriate tools.

Product, in bushels, on each of four fields of ten acres each.
Field A. Field B. Field C.

Field D.

5

50 45 40 35
10 150 140 130

120

15

270 255 240 255 [sic]
20 380 360 300

280

25

450 425 350 325
30 510 480 390

360

35

560 520 420 385
40 600 550 440

400

45

630 570 450 410
50 650 575 455

415

 

The following problems are based on the above table:—

Problem 1. Assuming that the labor of a man and team, with the appropriate tools, costs a farmer five dollars a day, and that corn is worth forty cents a bushel, how many days of such labor could he most profitably devote to the cultivation of each of the four fields?

Problem 2. Assuming that corn is worth only 33 1/3 cents a bushel, how much labor, etc., could he most profitably apply to the cultivation of each field,—the cost of labor, etc., remaining the same?

Problem 3. Assuming that a farmer has only 200 days’ labor to use, but that he can have rent free an indefinite amount of land of the grade of Field A, how much land could he most profitably use? How much land of the grade of Field C could he most profitably use?

Problem 4. How much land of each grade could he most profitably use if he had to pay five dollars an acre rent, corn being worth fifty cents a bushel, other conditions the same as in Problem 3?

Problem 5. Assuming that the two fields A and C are owned by the same farmer, and that he has but 20 days’ labor which he can devote to their cultivation, how could these 20 days be most profitably distributed among them? How could 25 days be most profitably distributed? 35 days? 50 days? 60 days? 70 days? 90 days?

 

Source: Harvard University Archives. Syllabi, course outlines and reading lists in economics, 1895-2003. Box 1. Folder “Economics, 1904-05”.

Image Source: Thomas Nixon Carver in Harvard Class Album 1906.

Categories
Exam Questions Harvard Sociology Suggested Reading Syllabus

Harvard. Sociology. Syllabus, reading assignments, final exam. Carver and Joslyn, 1927-28

 

This post has two functions: it adds to the syllabi for sociology taught at Harvard previously transcribed:  

Economics 3. Thomas Nixon Carver and William Z. Ripley, 1902
Economics 8. Thomas Nixon Carver, 1917-18.

It also serves as a meet an economics Ph.D. alumnus from Harvard post. The 1927-28 offering of Economics 8 was co-taught by Professor Carver and his sociology graduate student, Carl Smith Joslyn.

Carl Smith Joslyn (b. 20 Aug 1899 in Springfield, MA.; d. 23 Dec 1986 in Worthington, MA) went to Central High School in Springfield. At Harvard he received the Class of 1844 Scholarship (1919-1920). He went on to chair the sociology department at the University of Maryland, during which time he hired young C. Wright Mills.

________________

Carl Smith Joslyn
Harvard Ph.D. in Economics, 1930.

Carl Smith Joslyn, A.B. 1920
Subject, Economics. Special Field, Sociology. Thesis, “The Social Origins of American Business Leaders.” Instructor in Economics and Tutor in the Division of History, Government, and Economics, and Tutor in Sociology and Social Ethics, Harvard University.

Source: Harvard University. Report of the President of Harvard College, 1929-30. Page 120.

________________

Course Enrollment

[Economics] 8a1hf. Professor [Thomas Nixon] Carver and Mr. [Carl Smith] Joslyn.— Principles of Sociology

Total 79: 7 Graduates, 23 Seniors, 36 Juniors, 2 Sophomores 11 Other.

Source: Harvard University. Report of the President of Harvard College, 1927-28. Page 74.

________________

8. Principles of Sociology

[This is for 1928-29, virtually identical to 1924-25 description]

Mon., Wed., and (at the pleasure of the instructorFri., at 12.
Professor Carver and Mr. Joslyn

A study of human adaptation. Progress defined as adaptation. In what does progress consist, how may it be verified, what are the factors that promote or hinder it? The biological as well as the psychological, moral, economic, and political factors are studied. Attention is given to problems of moral adjustment and readjustment, of active control of the environmental factors, of economizing human energy and of social control.

Source: Division of History, Government, and Economics 1928-29.  Official Register of Harvard University, Vol. XXV, No. 29 (May 26, 1928), p. 68.

________________

Economics 8

I.
Introduction

  1. The Nature, Scope, and Method of Sociology

A study of purposeful human association.
Relation to Linguistics, Psychology, Jurisprudence, Ethics, Politics, Economics.

Carver, Sociology and Social Progress, pp. 1-14; 65-79.
Bushee, Principles of Sociology, ch. 1.

  1. The Evolutionary Concept in Sociology:
    (1) Continuity; (2) Change; (3) Differentiation; (4) Fixation.

Spencer, Principles of Sociology, Pt. I, ch. 1. Pt. II, chs. 1-4.
Bristol, Social Adaptation, pp. 29-40; 123-149.

  1. The Mechanism of Organic and Super-organic (Social) Evolution Compared.
    (1) Variation. (a) spontaneous or artificially produced; (b) minute or extreme.
    (2) Selection. (a) Natural. (b) social.

Bristol, Social Adaptation, pp. 55-79.
Carver, Sociology and Social Progress, pp. 276-299.
Bushee, Principles of Sociology, pp. 42-56.
Carver, Essays in Social Justice, pp. 1-27.

  1. The Origin and Development of Human Society.
    Survival value of (a) associated effort; (b) social inclination.

Giddings, Principles of Sociology, pp. 199-229; 256-323.
Dealey and Ward, Textbook of Sociology, Ch. I.

  1. The Nature and Conditions of Social Progress. Progress considered as the adaptation of the organism, man, to his environment: the method of adaptation being (a) Passive, or (b) active; the character of the environment being (a) physical, or (b) social.

Bushee, Principles of Sociology, pp. 19-41; 73-103.
Carver, Sociology and Social Progress, pp. 88-120.
Bristol, Social Adaptation, Preface and Introduction.

  1. The Limits of Social Progress: A mutual fitting together or balancebetween the passive and the active forms of adaptation.
    (1) on the physical side, (a) such modifications as will enable it to live healthfully in the modified physical environment, (b) such improvements of the physical environment as will so fit the modified human organism as to enable it to live healthfully.
    (2) on the moral side; (a) such modifications of the intellectual and moral nature of man as will cause individuals to react favorably to such stimuli as can be brought to bear upon them by an improved system of social control: (b) such improvements in the system of social control as will secure favorable responses from the improved intellectual and moral nature of man.

Bristol, Social Adaptation, pp. 221-304.

II.
A. Passive physical adaptation.

  1. Race and Environment as Factors in Social Progress.

Carver, Sociology and Social Progress, pp. 174-243; 498-500; 631-636.
Bristol, Social Adaptation, pp. 105-120.

  1. The Stability of the Racial Factor in Historic Time: the Inheritance of Acquired Characters.

Bushee, Principles of Sociology, pp. 362-368.
Popenoe & Johnson, Applied Eugenics, pp. 25-74; 99-115; 402-423.

  1. The Displacement of Natural Selection by Social Selection and its Consequences:
    (a) the Differential Birth-rate; (b) Philanthropy; (c) The Punishment of Criminals; (d) Military Selection.

Carver, Sociology and Social Progress, pp. 392-409; 647-653; 676-696.
Popenoe and Johnson, Applied Eugenics, pp. 116-146.
Bushee, Principles of Sociology, pp. 386-413.
Bristol, Social Adaptation, pp. 92-102.

  1. The Correlation of Ability and Social Status; Nature and Nurture in Social Stratification. Tests of Ability; (a) economic. (b) psychological.

Popenoe & Johnson, Applied Eugenics, pp. 1-24; 75-98.
Bushee, Principles of Sociology, pp. 326-361; 369-385.

  1. The Qualitative Control of Population; Eugenic and Dysgenic Factors in Modern Society.

Popenoe and Johnson, Applied Eugenics, pp. 176-279.

  1. The Increase of Population in Modern Times:
    a) General, (b) local, (c) occupational.

East, Mankind at the Crossroads, pp. 45-109; 146-198.

  1. The Quantitative Control of Population; the Operation of Positive and Preventive Checks in Modern Society.
    The Redistribution of population to relieve congestion. (a) local; (b) occupational.

Carver, Sociology and Social Progress, pp. 133-173.
East, Mankind at the Crossroads, pp. 231-283.

  1. Marriage and the Family; Disintegrative Forces and their Control.

Bushee, Principles of Sociology, pp. 252-273.
East, Mankind at the Crossroads, pp. 318-339.
Carver, Sociology and Social Progress, pp. 317-375; 674-675.

B. Passive Intellectual and Moral Adaptation.

  1. The Raw Material of Mental and Moral Development; Human Nature and its Re-Making

McDougall, Social Psychology, pp. 19-120.

  1. The Original Nature of Man; Instinct vs. Environment in Human Institutions.

McDougall, Social Psychology, pp. 121-227.

  1. The Psychology of the Crowd; Fundamental Processes of Social Behavior; the Nature of the “Group Mind”.

Carver, Sociology and Social Progress, pp. 503-521.
Bushee, Principles of Sociology, pp. 417-444.
McDougall, Social Psychology, pp. 279-301, 322-351.

  1. Education as the Instrument of Intellectual Adaptation; a Sociological View of the Objective and the Methods in Education.

Spencer, Education, pp. 21-128.

  1. Religion as the Instrument of Moral Adaptation; an Appraisal of Current Tendencies in Religion and Ethics.

Carver, Sociology and Social Progress, pp. 481-497.
Bushee, Principles of Sociology, pp. 529-549.
Carver, Religion Worth having, pp. 3-24; 93-140.

  1. The Problem of the Morally Unadapted; the Nature and Causes of Crime; a Program for Social Control.

Carver, Sociology and Social Progress, pp. 654-673.
Ferri, Criminal Sociology (to be assigned).
Parmelee, Criminology (to be assigned).

C. Active Physical Adaptation.

  1. Material Adaptation as the Productive Utilization of Human Energy; Prevalent Forms of Waste and their Elimination.

Carver, The Economy of Human Energy, pp. 140-181.
Veblen, The Theory of the Leisure Class, pp. 35-101.

  1. The Problem of Material Mal-Adaptation; Poverty and its Causes; a Program for Social Reform.

Carver, Essays in Social Justice, pp. 349-383.
Warner, American Charities, pp. 36-90.

  1. The Nature and Justification of Property; Problems of Ownership and Control in Modern Industry.

Carver, Essays in Social Justice, pp. 304-323.
Tawney, The Acquisitive Society, pp. 1-83.

  1. Radical Programs of Social Reform; Socialism, Anarchism, Syndicalism, and their Variants.

Carver, Essays in Social Justice, pp. 232-263.
Taussig, Inventors and Money-makers, pp. 76-135.

  1. Liberty and Equality as Practicable and Compatible Ideals; the Peculiar Destiny of the American Nation.

Carver, Essays in Social Justice, pp. 264-280.
Carver, The Present Economic Revolution in the United States, pp. 15-65; 233-263.

D. Active Moral Adaptation, or Social Control in its Broader Aspects.

  1. The Place of the State in Human Adaptation. Physical Compulsion as a System of Social Control. Punishment. Voluntary Agreement. The Problem of the Reconciliation of Group Interests and Individual Interests.

Bushee, Principles of Sociology, pp. 176-205.
Carver, Sociology and Social Progress, pp. 750-763.
Mill, Essay on Liberty, chs. 1, 2, and 4.

  1. The Essential Nature of Democracy; Sensitivity and how it is achieved (a) in a coercive state, (b) in a non-coercive business.

Spencer, Principles of Sociology, Pt. V, Chs. XVII, XVIII and XIX.

  1. Problems of Modern Democracy; a Survey of the claims of Democracy as the “Ideally Best Polity”

Carver, Sociology and Social Progress, pp. 764-787.
Mill, Essay on Representative Government, chs. 1, 2, 3, 5, and 6.

  1. The Possibility of Progress; a Recapitulation of Inorganic, Organic, and Social Evolutions and a Forecast of Future Developments.

(Reading to be assigned)

 

Reading Period

Ec 8a Professor Carver.

Sumner and Keller: Science of Society, Vol. I. Chs. I-X inclusive, Chs. XVIII, XIX.

Source: Harvard University Archives. Syllabi, course outlines and reading lists in economics, 1895-2003 (HUC 8522.2.1) Box 2, Folder “Economics, 1927-1928”.

________________

1927-28
HARVARD UNIVERSITY

ECONOMICS 8a1
Final Examination

Allow about one hour to each part of the examination.

I

  1. Below are given two contrasting views regarding: (a) the effects which an increase in numbers “in any given state of civilization” might be expected to have on the productive capacity of society; (b) the cause of want and misery in society. Which of these seems to you the more reasonable in each of these respects, and why? State in each case the considerations which, in your opinion, led the writer to take the particular view of the matter which he did.
    “A greater number of people cannot, in any given state of civilization, be collectively so well provided for as a smaller. The niggardliness of nature, not the injustice of society, is the cause of the penalty attached to over-population.”
    “I assert that in any given state of civilization a greater number of people can collectively be better provided for than a smaller. I assert that the injustice of society, not the niggardliness of nature, is the cause of the want and misery which the current theory attributes to over-population.”
  2. What is the attitude of Sumner and Keller on the question of “natural” rights? What is your own attitude? Would a man whose labor is absolutely superfluous to society have any right to a subsistence, in your opinion? Explain fully the grounds on which you base your judgment.

II

  1. Discuss the relation of sensitivity to democracy and point out the principal ways by which those who govern or manage are made sensitive to the interests of those who are governed or managed.
  2. What is meant by the vertical mobility of labor and what social institutions tend to decrease and what tend to increase it?
  3. Suppose that, from the beginning of human evolution, individual effort had been more effective than associated effort, do you think that men would have developed a social nature? Give reasons for your answer.

III

  1. Sumner and Keller have traced back all of our important social institutions to four primary interests in man. What are these interests and what are the institutions arising from each of them?
  2. Explain concisely each of the following terms, showing by your answer that you have a clear understanding of their several meanings:
    1. the man-land ratio;
    2. parallel induction;
    3. intellectual egalitarianism;
    4. maintenance-mores;
    5. ghost-fear;
    6. non-sustentative lethal selection;
    7. Marx’s theory of economic stratification;
    8. assortative mating
  3. Men are not sufficiently equipped with instincts to insure automatic behavior which has survival value in the complex life of modern society, neither are they sufficiently endowed with intelligence to secure rational behavior which has survival value. Between the limited field of behavior controlled by instinct and the equally limited field of behavior controlled by reason, there is apparently a wide gap. How is this gap filled?

 

Source: Harvard University Archives. Examination Papers Mid-years, 1927-1928(HUC 7000.55). Papers printed for Mid-year Examinations: History, History of Religions, …, Economics, …, Military Science, Naval Science. January-February, 1928.

________________

JOSLYN AWARDED $6000
END PRIZE FALLS TO PENN. MAN

May 22, 1920

Carl Smith Joslyn ’20 of Springfield, now working his way through college, has won the Truxton Beale prize of $6000. This award was made as a result of the Walker Blaine Beale memorial contest for a Republican Platform suitable for use in the approaching campaign. The prize was offered by Truxton Beale for the purpose of stimulating political study among young people, and was to be won by a Republican not over 25 years of age.

His Platform Decisive and Complete

Mr. Joslyn’s platform is a well-built and well-reasoned document, embracing nearly a score of the outstanding questions of the day. His Republican convictions are set forth with incisive moderation, which lends emphasis to every statement. He deals expeditiously with the various international and socialistic delusions; sets forth a peace program as clear as it is decisive; makes a quick analysis of the league of nations and puts well defined limits to its powers. The greater part of his platform is, however, devoted to domestic problems, beginning with the high cost of living and following its economic and sociological ramifications through the relations of labor and industry, production and economy, taxation, railroads, foreign trade and merchant marine. ment. He ends with the following paragraphs:

“The Republican party appeals to the people for their support on the stand which it has taken against the abuse of the executive power and for the preservation of the sovereignty and independence of the United States. Its principles and policies are all formulated by a liberal and constructive statesmanship. Its creed is one of undivided Americanism; one faith, one loyalty, one devotion–and these in the service of upbuilding and strengthening the great United States of America, the country which gave the world the ideals of liberty and justice and which has dedicated its future to their perpetuation and advancement.”

Other Prizes Also Fall to College Men

The second prize of $3000 goes to Howard B. Wilson of Philadelphia, a student at the University of Pennsylvania and the third of $1000 to W. P. Smith, a student at the University of Michigan. The judges were President Nicholas Murray Butler of Columbia University, former United States Senator Beveridge and former United States Ambassador David Jayne Hill.

Source: Archive of the Harvard Crimson, May 22, 1960.

________________

History of U. Maryland’s Sociology Department

Although classes began on this campus in October 1859, the first sociology course was not taught until fall semester 1919.  The course was “Elementary Sociology.”  From the time of this first course until 1935, when a separate Department of Sociology was established, all sociology courses were offered by the Economics Department. During the 1970s, the Sociology Department was restructured and Anthropology and Criminology became separate programs.  Today, the Sociology Department houses the Center for Innovation, Program for Society and the Environment, Maryland Time Use Laboratory, Center for Research on Military Organizations, Group Processes Lab and is affiliated with the Maryland Population Research Center.

Over the years, the sociology faculty has included many nationally and internationally renowned scholars.  In the 1920s, sociology courses were taught by George Peter Murdock, who later created the Human Relations Area Files.  In 1938, Logan Wilson, who later became the President of the University of Texas, joined the faculty for a few years.  C. Wright Mills, the author of The Power Elite, White Collar, and The Sociological Imagination, was a member of the faculty from 1941-1945.  The most renowned scholar on the faculty during the last quarter-century was Morris Rosenberg, the world’s foremost student of how social forces shape the self-esteem.

Since its founding, the Department has had eleven leaders: Theodore B. Manny, Carl Joslyn, Edward Gregory, Harold Hoffsommer, Robert Ellis, Kenneth C. W. Kammeyer, Jerald Hage, William Falk, Lee Hamilton, Suzanne Bianchi, and Reeve Vanneman. The current chair is Patricio Korzeniewicz.

Among the many people who have earned a degree from this department and subsequently achieved considerable recognition are William Form, the first person to hold a Ph.D. (1944) from this department; Parren Mitchell, who became a member of the U.S. House of Representatives; Adele Stamp, for whom the Stamp Student Union is named, and Charles Wellford of the Department of Criminology and Criminal Justice.

Source: University of Maryland, Department of Sociology. Webpage: “History of the Sociology Department”.

Image Source: Thomas Nixon Carver (left) and Carl Smith Joslyn (right) from the faculty photos in the Harvard Class Album 1932.